Descargar como pdf o txt
Descargar como pdf o txt
Está en la página 1de 50

Capı́tulo 1

Relatividad especial y el
espacio–tiempo de Minkowski

1.1. Introducción
La relatividad especial fue desarrollada por Einstein en 1905 [35, 36] como una manera
de reconciliar la electrodinámica de Maxwell con el principio de relatividad de Galileo,
que dice que las leyes de la fı́sica deben ser las mismas en todo sistema inercial. Esta teorı́a
está basada en dos postulados: el primero es precisamente el principio de relatividad, y el
segundo es la invariancia de la velocidad de la luz.
Aún cuando fue Einstein quién desarrolló la relatividad especial en su forma final,
desde fines del siglo XIX varios otros investigadores estaban ya tras su pista, aunque aún
sin tener una imagen completa. Entre ellos podemos mencionar desde luego a Hendrik
Lorentz y Henri Poincaré, además de algunos quizá menos recordados hoy en dı́a como
Woldemar Voigt y Joseph Larmor.
En la década de los 1860’s James Clerk Maxwell desarrolló la forma final de la teorı́a
electromagnética. Dicha teorı́a predecı́a nuevos fenómenos tales como la existencia de las
ondas electromagnéticas, pero parecı́a estar en conflicto con el principio de relatividad.
En particular, el hecho de que la fuerza debida a un campo magnético B ~ actuando sobre
una partı́cula con carga q estuviera dada en términos de la velocidad de la partı́cula ~v por
~ llevaba inmediatamente a preguntarse en que sistema de referencia
la expresión q(~v ⇥ B)
deberı́a medirse dicha velocidad. De manera similar, era claro que una carga en reposo
solo tenı́a campo eléctrico y no magnético, mientras que una carga en movimiento (una
corriente eléctrica) producı́a un campo magnético. Pero, ¿quién decidı́a si la carga estaba
en reposo o en movimiento?

1
2 CAPÍTULO 1. RELATIVIDAD ESPECIAL

Por otro lado, Maxwell habı́a encontrado que la velocidad de las ondas electromagnéti-
cas en el vacı́o c estaba dada en términos de la permitividad eléctrica ✏0 y la permeabilidad
p
magnética µ0 del vacı́o como c = 1/ ✏0 µ0 , que resultaba ser aproximadamente igual al
valor conocido de la velocidad de la luz (lo que lo llevó a concluir que la luz era una
onda electromagnética).1 Sin embargo, dado que de acuerdo a la fı́sica newtoniana esta
p
velocidad tendrı́a que depender del sistema de referencia, el valor de c = 1/ ✏0 µ0 deberı́a
ser válido únicamente en un sistema de referencia especı́fico.2
Los problemas que acabamos de mencionar, junto con la idea mecanicista que se
tenı́a en esa época de que un fenómeno ondulatorio como la luz deberı́a necesariamente
propagarse en un medio material, llevó a postular que los campos electromagnéticos eran
en realidad perturbaciones en un medio que permeaba todo el espacio conocido como el
“éter luminı́fero”.3 La electrodinámica de Maxwell se suponı́a entonces válida solo en el
sistema de referencia en el cual dicho éter se encontraba en reposo, y la velocidad de la luz
deberı́a medirse en ese mismo sistema de referencia. Maxwell mismo trabajó en posibles
modelos mecánicos para el éter, y Lorentz desarrolló toda una teorı́a sobre la dinámica
de las cargas eléctricas conocida como la “teorı́a del electrón”.4
La teorı́a del éter, sin embargo, presentaba una serie de problemas. El éter deberı́a ser
un fluido para poder llenar todo el espacio, incluso el espacio intermolecular en medios
1
Es interesante notar que el valor de la velocidad de la luz se conocı́a ya desde el siglo XVII con
muy buena aproximación. Fue medido por primera vez por el astrónomo danés Ole Roemer en 1676 a
partir de comparar las leyes de Kepler con el movimiento de las lunas galileanas de Júpiter (Io, Europa,
Ganı́medes y Calisto), que aparentemente se atrasaban o adelantaban dependiendo de la posición relativa
entre Júpiter y la Tierra. Roemer razonó que si la luz tenı́a una velocidad de propagación finita, tardarı́a
más en llegar cuando Júpiter estuviera más lejos, y utilizó este hecho para determinar el valor de dicha
velocidad [121, 127]. El valor que obtuvo Roemer era de aproximadamente c = 215, 000 km/s, un 75 % del
valor correcto de c = 299, 792.458 km/s. El error era debido principalmente a dos factores: la distancia de
la Tierra al Sol aún no se conocı́a con mucha precisión, y Roemer calculó que la luz tardaba 22 minutos
en atravesar la órbita terrestre, cuando en realidad tarda unos 16 minutos.
2
Hoy en dı́a el valor de la velocidad de la luz de c = 299, 792.458 km/s es exacto, esto es debido a
que la definición moderna de un metro es precisamente la distancia que recorre la luz en 1/299,792,458
segundos. El segundo, a su vez, se define ya no como una fracción del dı́a terrestre, sino a partir de la
transición entre dos niveles hiper-finos del átomo de Cesio 133.
3
La naturaleza ondulatoria de la luz fue propuesta por Robert Hooke en 1665 y por Christiaan
Huygens en 1678, y fue posteriormente confirmada experimentalmente por Thomas Young en 1803. La
existencia del éter de hecho fue propuesta ya por Huygens desde 1690, y solo muy posteriormente fue
adaptada a la electrodinámica de Maxwell.
4
La teorı́a del electrón de Lorentz, desarrollada a partir de 1892, no se refiere directamente a las
partı́culas elementales que hoy en dı́a conocemos como electrones y que fueron descubiertas por Joseph
John Thomson en 1897. Inicialmente la palabra “electrón” se referı́a más bien a hipotéticos “átomos de
electricidad”, y solo posteriormente fue asociada a la partı́cula descubierta por Thomson (el protón no
fue descubierto sino hasta 1909 por Ernest Rutherford).
1.1. INTRODUCCIÓN 3

como el aire o el vidrio, ya que la luz claramente podı́a propagarse en ellos. Por otro lado,
deberı́a ser un sólido para permitir ondas transversales como lo son las ondas electro-
magnéticas, y deberı́a ser enormemente rı́gido para soportar las altı́simas frecuencias que
podı́an tener dichas ondas. Deberı́a también tener cero viscosidad y masa extremadamente
baja, o de otra forma afectarı́a las órbitas de los planetas de manera visible. Todas estas
propiedades, algunas de ellas incluso contradictorias, hacı́an del éter algo casi imposible
de entender.
Pero el problema más grande vino con los experimentos de Albert Michelson y Edward
Morley. Como hemos mencionado, la velocidad de la luz deberı́a tener el valor c solo
en el sistema de referencia del éter en reposo, pero la Tierra se mueve alrededor del
Sol con una velocidad de unos 30 km/s, por lo que es claro que su velocidad relativa
respecto al éter deberı́a cambiar durante una órbita. Esto implica que la velocidad de
la luz medida en el laboratorio deberı́a variar a lo largo de un año. En 1881, Michelson
diseño un interferómetro para medir el cambio en la velocidad de la Tierra respecto al
éter. El experimento dio resultados negativos, pero el aparato aún no era muy preciso.
En 1887, Michelson y Morley repitieron el experimento con una versión mejorada del
interferómetro, obteniendo de nuevo resultados negativos: la velocidad de la luz era la
misma a lo largo del año, y cualquier variación que pudiera existir era mucho menor a
la predicha por la teorı́a del éter. El experimento de Michelson y Morley es, sin duda, el
experimento “fallido” más famoso de la historia.
Los resultados de Michelson y Morley pusieron a trabajar a los fı́sicos teóricos en
diversas posibles explicaciones. La mejor explicación antes de Einstein fue la debida a
George Fitzgerald y Hendrik Lorentz, quienes de manera independiente propusieron que
al moverse a travésp del éter los objetos se contraen en la dirección de movimiento por
un factor igual a 1 v 2 /c2 , con v la velocidad de movimiento respecto al éter y c la
velocidad de la luz. Dicha contracción de Lorentz–Fitzgerald explicaba perfectamente el
resultado negativo del experimento de Michelson y Morley. Lorentz argumentaba que esa
contracción era debida a un efecto del éter en las fuerzas intermoleculares, pero un me-
canismo dinámico que explicara porqué todos los objetos sufrirı́an exactamente la misma
contracción resultaba difı́cil de encontrar.
En 1905 Einstein dio el golpe final a la teorı́a del éter con la relatividad especial
(aunque algunos investigadores, incluyendo a Lorentz y Poincaré, siguieron hablando del
éter por varios años más). En esta nueva teorı́a la velocidad de la luz es la misma en todo
sistema inercial por construcción, y el concepto del éter se vuelve superfluo.5 El precio a
5
No es claro si Einstein sabı́a del experimento de Michelson y Morley en 1905. Lo que sı́ es claro es que
no los menciona en sus artı́culos originales. Al parecer en ese momento Einstein estaba más inspirado por
la elegancia del principio de relatividad y la teorı́a electromagnética de Maxwell que por los resultados
experimentales.
4 CAPÍTULO 1. RELATIVIDAD ESPECIAL

pagar es introducir una nueva cinemática distinta a la de Newton, donde los conceptos de
espacio y de tiempo dejan de ser absolutos.

1.2. Los postulados de Einstein


En su artı́culo original sobre la relatividad especial de 1905 [36] Einstein parte de dos
postulados fundamentales:

1. El principio de relatividad, según el cuál las leyes de la fı́sica son las mismas en todo
sistema inercial. Este principio tiene su origen en las observaciones de Galileo sobre
la imposibilidad de determinar el movimiento de un observador de manera absoluta,
y es completamente compatible con la mecánica newtoniana. Einstein, sin embargo,
lo eleva a un postulado acerca de todas las leyes de la fı́sica (en particular la óptica
y la electrodinámica), y no solo las leyes de la mecánica.6

2. La velocidad de la luz en el vacı́o es absoluta, es decir es la misma en todo sistema


inercial, y es independiente de la velocidad de la fuente de la luz (o del observador).

A primera vista podrı́a parecer que los dos postulados anteriores son contradictorios.
El primero parece indicar que no existen las velocidades absolutas, mientras que el se-
gundo nos dice que sı́ hay una: la velocidad de la luz. En su artı́culo original, Einstein
mismo hace mención de esta aparente contradicción. Pero notemos que en realidad el pri-
mer postulado no menciona en ningún momento velocidades (relativas o absolutas), sino
solamente la invariancia de las leyes de la fı́sica en los sistemas inerciales. Crucialmente, el
primer postulado escrito de esta forma resulta ser compatible con la existencia de una sola
velocidad absoluta, que de acuerdo al segundo postulado tomaremos como la velocidad
de la luz.
¿Por qué la velocidad de la luz es especial? Por dos motivos, uno empı́rico y otro
teórico: 1) el experimento de Michelson y Morley habı́a mostrado empı́ricamente que
esta velocidad era siempre la misma, sin importar el estado de movimiento de la Tierra,
y 2) las ecuaciones de Maxwell permitı́an derivar la velocidad de la luz sin hacer mención de
ningún sistema de referencia especı́fico (el éter se introducı́a a posteriori). Lo importante
6
Cabe mencionar que el primero que usó le nombre “principio de relatividad” fue de hecho Henri
Poincaré. En 1904 lo enuncia ya de una forma casi idéntica a la de Einstein [106]. La diferencia más
importante es que Poincaré y Lorentz veı́an a este principio como algo que tenı́a que demostrarse a partir
de la electrodinámica, mientras que Einstein lo toma como un postulado. De hecho, en 1909 Lorentz llega
a decir que “Einstein simplemente postula lo que nosotros hemos deducido, con cierta dificultad y no de
manera totalmente satisfactoria, a partir de las ecuaciones fundamentales del campo electromagnético”.
1.2. LOS POSTULADOS DE EINSTEIN 5

aquı́ es que existe una velocidad absoluta, que como veremos más adelante es la que
define las relaciones de causalidad en nuestro Universo. Otras interacciones, tales como
la gravedad o la interacción nuclear fuerte, también se propagan a esta misma velocidad,
ası́ que bien podrı́amos haberla llamado la “velocidad de la gravedad” o “la velocidad de los
gluones” (los portadores de la fuerza nuclear fuerte) de haber entendido estas interacciones
antes que la electromagnética. Es más, si el dı́a de mañana se descubre que existe una
partı́cula que se mueve ligeramente más rápido que la luz esto no destruirı́a a la teorı́a de
la relatividad, sino que nos llevarı́a a concluir que la velocidad absoluta no coincide con
la de la luz, y por lo tanto los fotones probablemente tendrı́an una pequeñı́sima masa.7

La relatividad especial hace énfasis en algo que quizá podrı́amos bautizar como el
“postulado cero”, y eso es precisamente el suponer que los sistemas de referencia inerciales
existen. De hecho, se mencionan en los dos postulados de Einstein. Pero, ¿qué es un sistema
inercial? En términos simples un sistema inercial se puede definir como aquel en el que se
cumple la primera ley de Newton: objetos libres de fuerzas externas se mantienen en un
estado de movimiento rectilı́neo uniforme. Aunque la definición anterior parece simple,
en realidad esconde ya una serie de complejidades. Para empezar, debemos poder definir
qué es una lı́nea recta, y que significa movimiento uniforme. La definición de un sistema
inercial entonces debe incluir alguna noción sobre las propiedades geométricas del espacio,
ası́ como una noción del tiempo. Vamos entonces a enunciar algunas de las propiedades
básicas que debe tener un sistema inercial:8

1. El espacio tridimensional tiene geometrı́a euclidiana, y esa estructura se mantiene


con el tiempo. Esto implica que sabemos definir lı́neas rectas, tenemos una noción
de paralelismo, y además sabemos medir distancias.

2. Las distancias entre dos puntos del espacio son independientes del tiempo. Alterna-
tivamente, podemos decir que dados dos objetos en reposo uno respecto al otro, y
libres de fuerzas externas, la distancia entre ellos se mantiene siempre igual.

3. El tiempo fluye al mismo ritmo en todos los puntos del espacio. Operativamente,
podemos situar una serie de relojes sincronizados en reposo unos respecto a otros
en distintos puntos del espacio, y estos permanecerán sincronizados para siempre.
7
Esto estuvo a punto de ocurrir cuando el experimento de neutrinos OPERA en la mina Gran Sasso
(en Italia) anunció en 2012 que habı́a medido una velocidad para los neutrinos provenientes del CERN (en
Suiza) mayor a la de la luz en una parte en 105 . El resultado finalmente resultó ser un error de medición.
8
Para una discusión muy detallada sobre los conceptos de espacio y tiempo en la teorı́a de Newton
ası́ como en la relatividad puede verse el libro de Tim Maudlin [80].
6 CAPÍTULO 1. RELATIVIDAD ESPECIAL

En el tercer punto que acabamos de mencionar hablamos de relojes sincronizados.


¿Cómo sincronizamos relojes en distintos puntos del espacio? Fácil, situamos una serie de
relojes en reposo entre sı́ a distancias conocidas, y luego a partir de un punto especı́fico,
que podemos llamar el “origen”, enviamos una señal a velocidad también conocida a un
tiempo que definimos como t = 0. Al recibir dicha señal, cada reloj ajusta su hora a t = T ,
con T el tiempo que toma a la señal recorrer la distancia del origen a dicho reloj. Nótese
que se puede usar cualquier señal que viaje a velocidad fija y conocida (una tortuga por
ejemplo), no tiene que ser un señal luminosa. Usar luz, sin embargo, tiene la ventaja de
que sabemos que su velocidad es siempre la misma de acuerdo al segundo postulado de
Einstein.
La primera ley de Newton implica ahora que si existe un sistema de referencia inercial,
entonces existe un número infinito de ellos moviéndose unos respecto a otros a velocidad
relativa constante en todas direcciones. Aquı́ es importante enfatizar que nuestra definición
de sistema inercial es de hecho bastante menos restrictiva que la de Newton. No hemos
hablado de un espacio o un tiempo absolutos, y no hemos dicho nada acerca de la relación
entre las distancias medidas en un sistema inercial y aquellas medidas en otro, o entre el
ritmo al que fluye el tiempo en los distintos sistemas inerciales.
Aunque no es parte de la definición, resulta conveniente introducir un sistema de
coordenadas cartesianas en nuestro espacio tridimensional euclidiano, y ası́ lo haremos de
ahora en adelante (aunque en capı́tulos posteriores consideraremos el caso de coordenadas
curvilı́neas).

¿Existen los sistemas inerciales en la naturaleza? La intuición nos dice que sı́, o que
por lo menos son una excelente aproximación a las propiedades del espacio y del tiempo a
las que estamos acostumbrados. De hecho, toda la fı́sica de Newton asume su existencia.
La relatividad especial también los toma como su punto de partida.
Aquı́ vale la pena preguntarse qué es lo que hace “especial” a la relatividad especial.
Es frecuente oı́r decir, incluso entre algunos fı́sicos, que la relatividad especial es especial
debido a que no puede describir objetos acelerados u observadores acelerados. Esto no
podrı́a estar más equivocado. La relatividad especial es en esencia una nueva cinemática,
y dentro de ella por supuesto que se puede hacer dinámica, es decir estudiar el efecto
de las fuerzas sobre los objetos fı́sicos, por lo que las aceleraciones aparecen todo el
tiempo. Observadores acelerados, o más correctamente sistemas de referencia acelerados,
también pueden incluirse (de hecho los estudiaremos en capı́tulos posteriores), aunque las
matemáticas se vuelven bastante más complejas. Lo que hace “especial” a la relatividad
especial es precisamente el hecho de que asume la existencia de los sistemas inerciales.
Como ya hemos mencionado, en esto la relatividad especial no difiere de la mecánica
newtoniana. Veremos más adelante que en la relatividad general los sistemas inerciales
1.3. UNIDADES GEOMÉTRICAS 7

globales ya no existen.
Un último comentario, ¿por qué se llama “teorı́a de la relatividad”? Este nombre
es quizá desafortunado, y se debe a que restaura la compatibilidad entre la teorı́a elec-
tromagnética de Maxwell y el principio de relatividad de Galileo, que aparentemente se
encontraban en conflicto, sin la necesidad de postular un sistema de referencia privilegiado
(el del éter). El nombre de hecho no se debe a Einstein, quién inicialmente la llamaba
“teorı́a invariante” (Invariantentheorie en alemán), haciendo énfasis en las cantidades que
permanecen iguales en todo sistema de referencia inercial. Fue Alfred Bucherer quién la
llamó teorı́a de la relatividad por primera vez en 1906, y este nombre se popularizó rápi-
damente (Lorentz se habı́a referido a ella antes como “teorı́a relativa”).9 Por desgracia, el
nombre se presta a que la gente diga con frecuencia que “Einstein demostró que todo es re-
lativo”, lo cuál por supuesto es absurdo. Si revisamos los postulados de Einstein, podemos
ver que ambos se refieren a cosas absolutas: las leyes de la fı́sica son absolutas (las mismas
en todo sistema inercial), y la velocidad de la luz es absoluta. De hecho, en la relatividad
siempre se hace énfasis en aquellas cantidades fı́sicas que resultan ser absolutas, es decir
aquellas que son iguales en todo sistema de referencia. La teorı́a quizá deberı́a haberse lla-
mado “teorı́a de la velocidad de la luz absoluta”, lo que desde luego es un nombre mucho
menos elegante y llamativo. Aunque en realidad, como veremos más adelante, la manera
más correcta de referirse a esta teorı́a es simplemente decir que estamos trabajando en el
espacio–tiempo de Minkowski.

1.3. Unidades geométricas


Antes de seguir adelante con las consecuencias de los postulados de Einstein, haremos
un breve paréntesis para hablar un poco acerca de las unidades. En el sistema interna-
cional las distancias se miden en metros y el tiempo se mide en segundos. El metro se
define originalmente como una fracción de la circunferencia terrestre (1/40, 000, 000), y el
segundo como una fracción del dı́a terrestre, aunque las definiciones modernas se basan
en la velocidad de la luz y en la frecuencia de ciertas transiciones hiper-finas del átomo de
Cesio. En todo caso, las definiciones son arbitrarias, y en términos de ellas la velocidad de
la luz es un número muy grande, c = 299, 792, 458 m/s, y por lo mismo difı́cil de visua-
lizar de manera intuitiva. La velocidad de la luz es simplemente demasiado grande para
nuestra experiencia cotidiana. Debido a esto, en la relatividad resulta muy conveniente
redefinir nuestras unidades de manera que la velocidad de la luz se vuelva un número más
manejable.
9
Es interesante notar que Poincaré nunca reconoció las contribuciones cruciales de Einstein, y se
referı́a siempre a esta teorı́a como “la nueva mecánica de Lorentz”.
8 CAPÍTULO 1. RELATIVIDAD ESPECIAL

A partir de este momento vamos entonces a tomar la velocidad de la luz siempre


igual a uno, c = 1. Para lograr esto resulta necesario cambiar la unidad de tiempo: de
ahora en adelante ya no vamos a medir el tiempo en segundos, sino que lo mediremos en
metros, con un metro de tiempo definido como el tiempo que le toma a la luz recorrer una
distancia de un metro (equivalente a 3.3356 ⇥ 10 9 segundos). A estas nuevas unidades en
las que tomamos c = 1 se les conoce en general como unidades geométricas.10 En unidades
geométricas resulta mucho más fácil pensar en objetos con velocidades cercanas a la de la
luz, y en particular hacer diagramas sobre este tipo de situaciones (ver siguiente sección).
El haber redefinido la unidad de tiempo para que sea igual a la de distancia tiene
varias consecuencias importantes. Primero que nada, desde ahora p la velocidad se vuelve
una cantidad adimensional, por lo que expresiones tales como 1 v 2 tienen sentido. Por
otro lado, cualquier unidad derivada que incluya unidades de tiempo se ve alterada. Por
ejemplo, la aceleración ahora se medirá en m 1 , y la energı́a se medirá en kilogramos (y
ya no en Joules). Otra consecuencia importante es que en nuestras expresiones ya no va
a aparecer la velocidad de la luz (dado que vale 1). Por ejemplo, en lugar de E = mc2
vamos a tener simplemente E = m. Cuando sea necesario siempre podemos reintroducir
los factores correctos de c mediante un simple análisis dimensional.

1.4. Diagramas espacio–tiempo


Cuando se estudia relatividad especial resulta particularmente útil hacer diagramas
que muchas veces ayudan a aclarar situaciones que podrı́an resultar confusas. Comúnmen-
te, vamos a considerar movimiento únicamente en una sola dimensión espacial, a la que
llamaremos x por conveniencia. Nuestros diagramas considerarán entonces la posición x
y el tiempo t, por lo que serán bidimensionales. Por convención tomaremos siempre el eje
espacial x como horizontal, y el eje temporal t como vertical.
A un punto (t, x) en este diagrama espacio–tiempo le llamaremos un evento, algo
que ocurre en un lugar especı́fico a un tiempo dado. Por convención también siempre
colocaremos la coordenada temporal antes que la espacial: (t, x) y no (x, t). A la trayectoria
que sigue un objeto en este diagrama espacio–tiempo le llamaremos la lı́nea universo del
objeto (a veces también conocida como la “lı́nea mundo”). Finalmente, dado que en
nuestras unidades la velocidad de la luz es igual a 1, la lı́nea universo de un haz de
luz, o un fotón, corresponde siempre a una lı́nea recta a 45 (inclinada a la derecha o a
10
Más adelante, cuando estudiemos relatividad general, también tomaremos la constante de la gravi-
tación de Newton igual a uno (G = 1), para lo cuál será necesario redefinir la unidad de masa. Estas
son realmente las unidades geométricas completas. Pero de momento solo vamos a modificar la unidad
de tiempo.
1.4. DIAGRAMAS ESPACIO–TIEMPO 9

t v<1
Haz de luz (v = 1)

Evento

v>1

45o

x
Figura 1.1: Diagrama espacio–tiempo. Un punto en el diagrama se denomina un evento. Lı́neas
rectas corresponden a la lı́nea universo de objetos que se mueven a velocidad constante. La luz
(fotones) se mueve en lı́neas rectas a 45 .

la izquierda dependiendo de la dirección de propagación). Todos estos conceptos están


representados en la Figura 1.1.
Un objeto que se mueve a velocidad constante en el diagrama espacio–tiempo tiene
como lı́nea universo una lı́nea recta, mientras que la lı́nea universo de un objeto que
se mueve con velocidad variable será en general curva. Como veremos más adelante, los
objetos con masa se mueven siempre más lento que la luz, por lo que la pendiente de su
lı́nea universo siempre debe ser mayor a uno. Un concepto particularmente importante
es el llamado cono de luz asociado a un evento. El cono de luz corresponde a todas las
posibles trayectorias de fotones que salen desde ese evento hacia el futuro, más todas
aquellas trayectorias de fotones que convergen en ese evento desde el pasado.11 Notemos
que el cono de luz está asociado a un evento especı́fico, y no a una lı́nea universo. Debido
a la invariancia de la velocidad de la luz el cono de luz es absoluto, es decir todos los
sistemas de referencia están de acuerdo en cuál es el cono de luz de un evento dado. Se le
llama “cono” debido a que si consideramos dos dimensiones de espacio y una de tiempo
las trayectorias de la luz que salen de un evento forman claramente un cono a 45 , o
estrictamente un cono doble debido a que también consideramos las trayectorias de la luz

11
De aquı́ en adelante usaremos la palabra “fotón” simplemente para referirnos a un haz de luz.
La naturaleza cuántica de la luz no es algo que nos preocupe tanto como las trayectorias de la luz en
el espacio–tiempo, aunque sı́ tendremos algo que decir más adelante sobre algunas propiedades de los
fotones.
10 CAPÍTULO 1. RELATIVIDAD ESPECIAL

t
Cono de luz

Evento (t,x,y,z)

x
Línea universo
Figura 1.2: Lı́nea universo de un objeto que se mueve con velocidad variable siempre menor que
la de la luz, y cono de luz asociado al evento (t, x, y, z). Nótese que el cono de luz se asocia a un
evento especı́fico, y no a toda la lı́nea universo.

que llegan desde el pasado hasta ese evento (ver Figura 1.2). Dado que el espacio en el
que vivimos tiene tres dimensiones, en realidad de trata de un hiper–cono en un espacio–
tiempo de cuatro dimensiones, pero eso no se puede representar de manera gráfica.

Antes de concluir esta sección vamos a introducir algo de notación. Las coordenadas
de un evento en el espacio–tiempo de cuatro dimensiones se denotarán por (t, x, y, z),
o de manera enteramente equivalente usando superı́ndices como (x0 , x1 , x2 , x3 ). Enfati-
zamos que se trata de superı́ndices y no de potencias, y por convención tomaremos el
ı́ndice 0 como el correspondiente a la coordenada temporal (la razón por la que usamos
superı́ndices y no subı́ndices quedará clara más adelante). Cuando queramos referirnos
a una coordenada arbitraria utilizaremos ı́ndices griegos x↵ , donde ↵ puede tomar cual-
quiera de los cuatro valores (0, 1, 2, 3). En algunas ocasiones necesitaremos referirnos solo
a las coordenadas espaciales, en cuyo caso utilizaremos ı́ndices latinos xi , donde ahora i
solo puede tomar los valores (1, 2, 3).12

12
Esta convención es muy común hoy en dı́a, pero de ninguna manera es universal. Por ejemplo,
algunos autores toman la coordenada temporal como t = x4 , por lo que las coordenadas de un evento
son (x1 , x2 , x3 , x4 ). Otros pueden utilizar ı́ndices latinos mayúsculos para las coordenadas en el espacio–
tiempo, y minúsculos para las coordenadas espaciales. Siempre es importante revisar las convenciones
que se utilizan en las distintas referencias.
1.5. LAS TRANSFORMACIONES DE LORENTZ 11

1.5. Las transformaciones de Lorentz


Uno de los resultados más importantes de la relatividad especial es el relacionado
con las transformaciones de coordenadas entre dos sistemas inerciales que se mueven uno
respecto al otro. Consideremos entonces un sistema inercial que denotaremos por O (y que
normalmente asociaremos al sistema del laboratorio), y un segundo sistema de referencia
Ō que se mueve respecto al primero con velocidad constante v a lo largo del eje x. En la
fı́sica newtoniana, la transformación de coordenadas entre ambos sistemas de referencia
está dada por las llamadas transformaciones de Galileo:
t̄ = t , x̄ = x vt , ȳ = y , z̄ = z . (1.5.1)
Todas las leyes de la mecánica newtoniana son invariantes ante esta transformación de
coordenadas. Sin embargo, las leyes de la electrodinámica de Maxwell no lo son, y esto
es lo que motivó a los fı́sicos del siglo XIX, incluyendo al propio Maxwell, a suponer que
estas leyes eran válidas solo en un sistema de referencia privilegiado que se asociaba al
éter luminı́fero.
Por otro lado, aún cuando suponı́an que el éter era real, muchos investigadores se
preguntaron si existı́an algunas transformaciones de coordenadas abstractas que dejaran
invariantes a las ecuaciones de Maxwell. En 1887 Woldemar Voigt encontró transforma-
ciones de coordenadas que dejaban invariante a la ecuación de onda y a la velocidad de
la luz, sin embargo dichas transformaciones no estaban aún completas pues la ecuación
de onda tiene además una invariancia de escala. Durante los siguientes años Lorentz, Lar-
mor y Poincaré avanzaron poco a poco en el desarrollo de las nuevas transformaciones.
La forma final fue encontrada por Lorentz en 1904. Cabe mencionar, sin embargo, que
dichas transformaciones eran abstractas y su sentido fı́sico no se entendı́a aún del todo.
Lorentz, por ejemplo, asumı́a que el éter era real (o que por lo menos existı́a un sistema
de referencia privilegiado), y que las coordenadas transformadas simplemente resultaban
útiles para hacer cálculos en sistemas que se movı́an con respecto al éter.
Fue finalmente Einstein quien comprendió que estas “transformaciones de Lorentz”,
como las habı́an bautizado Larmor y Poincaré, no eran un simple artefacto matemático
para facilitar los cálculos en sistemas electrodinámicos en movimiento respecto al éter,
sino que en realidad no era necesario suponer la existencia del éter y las transformaciones
de Lorentz eran las transformaciones correctas de coordenadas entre sistemas inerciales:
no existı́a un sistema de referencia privilegiado. Además, Einstein logró obtener estas
transformaciones de manera muy simple a partir de sus dos postulados, contrastando con
los complejos cálculos matemáticos que habı́an sido necesarios hasta entonces partiendo
de las ecuaciones de Maxwell.13
13
La derivación de las transformaciones de Lorentz partiendo de las ecuaciones de Maxwell no es nada
12 CAPÍTULO 1. RELATIVIDAD ESPECIAL

El salto conceptual de Einstein es crucial, pues si las transformaciones de Lorentz son


las correctas esto implica, primero, que la electrodinámica de Maxwell sı́ es perfectamente
compatible con el principio de relatividad tal y como está, y segundo y mucho más impor-
tante, que las transformaciones de Galileo no son las correctas, y por lo mismo tampoco
lo es la mecánica de Newton, ni los conceptos newtonianos de espacio y tiempo. Esto es
revolucionario.

Vamos a proceder ahora a encontrar la forma de la transformación de coordenadas a


partir de los postulados de Einstein. Por simplicidad asumimos que el sistema de refe-
rencia Ō se mueve respecto al sistema O con velocidad v a lo largo del eje x. El primer
paso es notar que, dado que el espacio–tiempo asociado a un sistema inercial es por cons-
trucción homogéneo e isotrópico (es decir igual en todos lados y en todas direcciones), las
transformaciones de coordenadas entre dos sistemas inerciales deben ser lineales. Esto se
debe a que, de no ser lineales, partiendo de una distancia x y un intervalo de tiempo t
en el sistema O obtendrı́amos valores de x̄ y t̄ en el sistema Ō que dependerı́an de la
posición inicial (t, x), y esto no es compatible con que el espacio–tiempo sea homogéneo
e isotrópico. Partimos entonces de una transformación de coordenadas lineal de la forma:
t̄ = ax + bt , (1.5.2)
x̄ = cx + dt , (1.5.3)
donde (a, b, c, d) son constantes a determinar. Al decir que son constantes nos referimos
a que no dependen ni de la posición, ni del tiempo, aunque sı́ dependen de la velocidad
v. En la expresión anterior hemos supuesto, por simplicidad, que los orı́genes de ambos
sistemas de referencia coinciden, es decir el evento (t = 0, x = 0) corresponde al evento
(t̄ = 0, x̄ = 0).
Consideremos ahora las consecuencias del segundo postulado de Einstein. Pensemos de
momento en un haz de luz propagándose hacia la derecha que se emite desde el origen. En
unidades geométricas esto implica que su trayectoria corresponde a x = t. Ahora, por la
invariancia de la velocidad de la luz, esto debe implicar también que x̄ = t̄. Sustituyendo
en las ecuaciones anteriores encontramos:
a+b=c+d. (1.5.4)
Si ahora consideramos un haz de luz moviéndose a la izquierda tenemos x = t, y la
invariancia de la velocidad de la luz implica que x̄ = t̄. Encontramos entonces:
a b=d c. (1.5.5)
trivial. En particular porque, como veremos más adelante, al pasar de un sistema inercial a otro los
campos eléctrico y magnético se mezclan entre sı́.
1.5. LAS TRANSFORMACIONES DE LORENTZ 13

Estos dos resultados implican que d = a y c = b, por lo que la transformación de coorde-


nadas se reduce a:

t̄ = ax + bt , (1.5.6)
x̄ = bx + at . (1.5.7)

Ahora, dado que por construcción el sistema Ō se mueve respecto a O con velocidad
v, esto implica que la posición del origen x̄ = 0 debe corresponder a la lı́nea x = vt. A
partir de esta observación encontramos que a = bv, y la transformación toma la forma:

t̄ = b (t vx) , (1.5.8)
x̄ = b (x vt) . (1.5.9)

Nótese que ahora tenemos solo una última constante por determinar. Por otro lado, es ya
claro que estas transformaciones no se reducen a las de Galileo para ningún valor de b.
Para encontrar el valor de la constante b invertimos la transformación. Un poco de
álgebra nos lleva a:
1
t= (t̄ + vx̄) , (1.5.10)
b(1 v2)
1
x= (x̄ + v t̄) . (1.5.11)
b(1 v2)
Pero dado que ambos sistemas de referencia son igualmente válidos, las transformaciones
de uno a otro deben tener la misma forma salvo por el signo de la velocidad v. Esto
implica entonces que:
1 1
b= 2
=) b2 = , (1.5.12)
b(1 v ) 1 v2
y finalmente:
1
b= p ⌘ , (1.5.13)
1 v2
p
donde hemos definido el llamado factor de Lorentz como := 1/ 1 v 2 . La transforma-
ción final de coordenadas toma la forma final:

t̄ = (t vx) , (1.5.14)
x̄ = (x vt) , (1.5.15)
ȳ = y , (1.5.16)
z̄ = z , (1.5.17)
14 CAPÍTULO 1. RELATIVIDAD ESPECIAL

donde agregamos ya las transformaciones (triviales) de y y z. Estas son las transformacio-


nes de Lorentz (asumiendo movimiento únicamente en el eje x).14 Es importante enfatizar
que en la relatividad estas son las transformaciones correctas entre dos sistemas inercia-
les, es decir aquellas que son compatibles con la invariancia de la velocidad de la luz. Las
transformaciones de Lorentz son claramente simétricas en x y t, lo que no ocurrı́a con
las transformaciones de Galileo. Esto es algo que ocurre con frecuencia en la relatividad:
obtenemos nuevas simetrı́as que no existı́an en la mecánica de Newton. Podemos notar,
además, que las transformaciones de Lorentz ya indican que algo extraño ocurre si permi-
timos que un sistema de referencia se mueva respecto a otro a velocidad mayor que la de
la luz: las transformaciones se vuelven complejas. Esto es una primera indicación de que
en la relatividad la velocidad de la luz no solo es absoluta, sino que también es la máxima
posible.
¿Cómo se relacionan las transformaciones de Lorentz con las de Galileo? Para ver esto
de momento vamos a reintroducir los factores correctos de c en las transformaciones de
Lorentz:

x̄ = (x vt) , (1.5.18)
t̄ = t vx/c2 , (1.5.19)
p
con = 1/ 1 v 2 /c2 . Si asumimos ahora que la velocidad v es muy baja comparada con
la velocidad de la luz |v| ⌧ c, o equivalentemente si tomamos el lı́mite cuando la velocidad
de la luz c se hace infinita, encontramos que = 1, y las transformaciones anteriores se
reducen a las de Galileo.15

Vamos ahora a reescribir las transformaciones de Lorentz de manera más compacta.


Para ello definimos la matriz jacobiana de la transformación como:
¯
¯ @x
⇤↵ := , (1.5.20)
@x↵
¯
donde x↵ se refiere a las coordenadas en el sistema O, y x a las coordenadas en Ō. Nótese la
¯
posición de los ı́ndices en ⇤↵ : el subı́ndice ↵ nos dice respecto a quién derivamos, mientras
que el superı́ndice ¯ nos dice a quién derivamos. Esto se vuelve más claro si introducimos
14
Las transformaciones considerando movimiento en una dirección arbitraria no son difı́ciles de encon-
trar, pero son bastante más complicadas y no particularmente interesantes.
15
La manera más sencilla de reducir expresiones relativistas a sus versiones newtonianas es en general
reintroducir los factores adecuados de c (mediante análisis dimensional), y simplemente tomar el lı́mite
c ! 1. Si, por otro lado, lo que se busca son las primeras correcciones a las ecuaciones newtonianas, se
debe asumir que v/c ⌧ 1 y conservar términos lineales en v/c.
1.5. LAS TRANSFORMACIONES DE LORENTZ 15

algo de notación adicional. De ahora en adelante vamos a abreviar la derivada parcial de


una función f de la siguiente manera:
@f
⌘ @↵ f ⌘ f,↵ . (1.5.21)
@x↵
Las tres expresiones denotan exactamente lo mismo. Aunque aquı́ en general vamos a
preferir usar @↵ f , es importante mencionar que la notación f,↵ (indicando la derivada
parcial simplemente por una coma) es también muy frecuente. En términos de esta nueva
notación la matriz jacobiana de la transformación de Lorentz está dada por:
¯ ¯
⇤↵ = @ ↵ x . (1.5.22)
Vemos ahora que la posición de los ı́ndices es consistente en ambos lados de esta expresión.
El ı́ndice que estaba abajo (subı́ndice) permanece abajo, y el que estaba arriba (superı́ndi-
ce) permanece arriba. La posición de los ı́ndices es muy importante en el cálculo tensorial
que veremos en los siguientes capı́tulos.
Sustituyendo las transformaciones de Lorentz en la definición de la matriz jacobiana
encontramos: 0 1
v 0 0
B v 0 0 C
⇤ := B@ 0
C. (1.5.23)
0 1 0 A
0 0 0 1
En términos de las componentes de esta matriz, la transformación de Lorentz se puede
escribir simplemente como:
X 3
¯ ¯
x = ⇤ ↵ x↵ . (1.5.24)
↵=0
Vamos a simplificar aún más la expresión anterior introduciendo la llamada “conven-
ción de la suma” (debida a Einstein): cuando en un mismo término aparece un ı́ndice
repetido, una vez como subı́ndice y otra como superı́ndice, se sobreentiende que se debe
sumar sobre todos sus valores permitidos. Usando esta convención la expresión para la
transformación de Lorentz toma la forma compacta:
¯ ¯
x = ⇤↵ x↵ . (1.5.25)
Un ı́ndice que se utiliza para indicar una suma, como el ı́ndice ↵ en la expresión
anterior, se conoce como un ı́ndice mudo. Los ı́ndices mudos se pueden renombrar sin
¯ ¯
afectar la expresión en absoluto. Por ejemplo las expresiones ⇤↵ x↵ y ⇤µ xµ son idénticas.
Cuando estudiemos cálculo tensorial usaremos frecuentemente este “juego” de renombrar
los ı́ndices mudos para poder simplificar expresiones largas.
16 CAPÍTULO 1. RELATIVIDAD ESPECIAL

1.6. Simultaneidad y causalidad


Una de las consecuencias más inmediatas de las transformaciones de Lorentz, y quizá de
las menos intuitivas, es la relatividad de la simultaneidad. Pero, ¿qué significa simulta-
neidad? Para aclararlo regresemos a nuestra definición de un sistema inercial. En esta
definición pedimos que el tiempo fluya al mismo ritmo en todos lados medido por relo-
jes en reposo en ese sistema de referencia. Esos relojes en reposo pueden sincronizarse
mediante señales luminosas, y una vez sincronizados permanecen sincronizados. Decimos
entonces que dos eventos son simultáneos si ocurren en distintos lugares del espacio, pero
a un mismo tiempo medido por dichos relojes sincronizados. En la mecánica de Newton,
dado que el tiempo es absoluto, esta definición de simultaneidad es también absoluta: si
dos eventos son simultáneos es un sistema inercial, lo son en todos. Pero en la relatividad
ya no ocurre ası́.
Para entender esto consideremos las transformaciones de Lorentz. Pensemos primero
en todos aquellos eventos que ocurren no a un mismo tiempo, sino en una misma posición
x̄ = 0 en el sistema Ō, es decir aquellos eventos que definen el eje t̄. Sustituyendo x̄ = 0
en la transformación de Lorentz para x̄, ecuación (1.5.15), obtenemos que en el sistema
de referencia O se cumple:
x = vt . (1.6.1)
Es decir, encontramos que el eje x̄ visto desde el sistema O corresponde a una lı́nea recta
que pasa por el origen con pendiente 1/v (t = x/v). Esto por supuesto no es ninguna
sorpresa, y simplemente nos dice lo que ya sabemos, que el sistema Ō se mueve respecto
al sistema O con velocidad v. Galileo y Newton habrı́an llegado a la misma conclusión.
Pero consideremos ahora aquellos eventos que ocurren a un mismo tiempo t̄ = 0 en
el sistema Ō, es decir los eventos que definen el eje x̄. Sustituyendo ahora t̄ = 0 en la
transformación de Lorentz para t̄, ecuación (1.5.14), encontramos:

t = vx . (1.6.2)

Esta es de nuevo la ecuación de una lı́nea recta que pasa por el origen, pero ahora con
pendiente v, y claramente no coincide con el eje x para v 6= 0. Es decir, todos aquellos
eventos que ocurren a un mismo tiempo t̄ = 0 y por lo tanto son simultáneos en el
sistema Ō, ocurren a distintos tiempos t y por lo tanto no son simultáneos en el sistema
O. Concluimos entonces que la simultaneidad no es absoluta, y su definición depende del
sistema de referencia en que estemos trabajando. Galileo y Newton no habrı́an estado de
acuerdo con esto, pero nos obligan las transformaciones de Lorentz.
La Figura 1.3 muestra en el panel izquierdo la posición de los ejes x̄ y t̄ vistos en
el sistema de referencia O (asumiendo v > 0), ası́ como las lı́neas con x̄ y t̄ constantes.
1.6. SIMULTANEIDAD Y CAUSALIDAD 17

- -
t t t t

q q

-
x
q -
x x
q
x
Figura 1.3: Ante una transformación de Lorentz los ejes coordenados rotan un ángulo
✓ = arctan(v) en direcciones opuestas. Panel izquierdo: sistema Ō visto desde O (asumiendo
v > 0). Panel derecho: sistema O visto desde Ō.

En el panel derecho se muestra la situación opuesta, los ejes x y t y las lı́neas de x y t


constantes vistas desde Ō. Podemos notar como una transformación de Lorentz rota los
ejes coordenados un mismo ángulo ✓ = arctan(v), pero el eje espacial y temporal rotan
en direcciones opuestas.
A todos los eventos que ocurren a un tiempo fijo en un sistema de referencia especı́fico
se les conoce como las superficies de simultaneidad (en realidad son hipersuperficies ya
que tienen tres dimensiones), y corresponden a lo que cada sistema de referencia considera
como el “espacio tridimensional” a un tiempo fijo. Notemos que, dado que las superficies
de simultaneidad no coinciden en los distintos sistemas de referencia, lo que un sistema de
referencia considera el “espacio” a un tiempo dado resulta ser una combinación de espacio
y tiempo en otro sistema de referencia: espacio y tiempo se mezclan al cambiar de sistema
de referencia.

¿El que la simultaneidad no sea absoluta representa un problema fı́sico? En realidad


no, el decir que dos eventos que ocurren en distintos lugares del espacio son simultáneos
es simplemente una convención y no tiene en realidad significado fı́sico. Desde luego, en
un sistema de referencia inercial especı́fico la simultaneidad es una convención que puede
resultar muy útil pues nos permite ordenar los distintos eventos en el tiempo (veremos
más adelante que en sistemas de referencia no inerciales el concepto de simultaneidad
deja de ser particularmente útil). De cualquier manera, en escalas pequeñas no hay mayor
problema, pues incluso si dos sistemas de referencia se mueven uno respecto a otro a casi
la velocidad de la luz, su definición de simultaneidad para eventos separados una distancia
18 CAPÍTULO 1. RELATIVIDAD ESPECIAL

-
t=0

A B t=0

-
t=0

Figura 1.4: Eventos A y B vistos desde tres sistemas de referencia inerciales. En el sistema
¯ tenemos
O tenemos tA = tB , en el sistema Ō tenemos t̄A > t̄B , y finalmente en el sistema Ō
¯ ¯
t̄A < t̄B .

D difiere a lo más en un tiempo t = D/c. Para el caso de la Tierra, por ejemplo, ese
tiempo resulta ser de solo 4 centésimas de segundo de un extremo al otro del planeta.
Incluso en el caso de la distancia a la Luna, la diferencia es de apenas poco más de un
segundo, por lo que no hay gran ambigüedad en el concepto de simultaneidad en el sistema
Tierra–Luna.
Cuando hablamos de distancias astronómicas, sin embargo, la ambigüedad en definir
la simultaneidad puede ser desde unos años para las estrellas cercanas, hasta muchos
millones de años cuando consideramos otras galaxias. Vemos ası́ que en realidad no tiene
ningún sentido preguntarse algo como: ¿qué estará pasando en la galaxia de Andrómeda en
este momento? Andrómeda está a dos millones de años luz de nosotros. A esa distancia
incluso alguien moviéndose en un avión a 1000 km/hora no estarı́a de acuerdo con la
definición de simultaneidad de una persona en reposo en el suelo por varios años.
Lo que realmente nos interesa no es si dos eventos ocurren “a un mismo tiempo”,
sino si puede existir o no una relación causal entre ellos. Y aquı́ aparentemente tenemos
un problema. Consideremos de momento tres sistemas de referencia inerciales: el sistema
del laboratorio O, un sistema Ō que se mueve respecto a O con velocidad v > 0, y un
tercer sistema Ō¯ que se mueve respecto a O con velocidad v. Y supongamos ahora que
tenemos dos eventos A y B que ocurren a un mismo tiempo t = 0 en el sistema O, como
se muestra en la Figura 1.4. De la Figura vemos claramente que en el sistema O se cumple
tA = tB = 0, por lo que ambos eventos son simultáneos. Por otro lado, para el sistema Ō
tenemos t̄A > 0 y t̄B < 0, por lo que en este sistema los eventos no son simultáneos, y B
¯ tenemos t̄¯ < 0 y t̄¯ > 0, por lo que en
ocurrió antes que A. Finalmente, en el sistema Ō A B
este sistema los eventos tampoco son simultáneos, pero ahora A ocurrió antes que B.
1.7. INVARIANCIA DEL INTERVALO 19

¿Quién tiene la razón? ¿Cuál evento ocurrió primero? La respuesta es que todos tie-
nen razón, y el orden temporal de los eventos A y B no esta bien definido, es cuestión de
convención. Pero, si el orden temporal de dos eventos no está bien definido, ¿no tenemos
ahora un problema potencial con la causalidad? La respuesta es que no, siempre y cuando
podamos garantizar que los eventos A y B no tienen relación causal. ¿Y cómo lo garanti-
zamos? Es aquı́ donde entra el cono de luz. Si dos eventos están dentro de sus respectivos
conos de luz (o sobre las fronteras de dichos conos), entonces todos los sistemas inerciales
estarán de acuerdo en el orden temporal de ambos eventos, por lo que en principio puede
existir una relación causal entre ellos (ver Ejercicio 1.3). Si, por el contrario, los eventos
están fuera de sus conos de luz, el orden temporal no está definido y dependerá del siste-
ma de referencia, por lo que no puede haber una relación causal entre ellos. Esta relación
siempre es recı́proca: si el evento A está dentro del cono de luz futuro (pasado) de B,
entonces B estará dentro del cono de luz pasado (futuro) de A.
Recordemos que el cono de luz es absoluto debido a que la velocidad de la luz es
absoluta. Todos los sistemas inerciales están de acuerdo en cuál es el cono de luz de
un evento dado. El cono de luz es la única estructura absoluta que podemos usar para
definir la causalidad. La conclusión es entonces la siguiente: si queremos tener relaciones
de causalidad bien definidas, donde las causas siempre ocurran antes que los efectos en
todo sistema de referencia, entonces debemos restringir todas las interacciones fı́sicas al
cono de luz. Dicho de otra forma, para rescatar la causalidad es necesario exigir que
ningún tipo de interacción fı́sica se propague más rápido que la luz. En particular ningún
objeto puede viajar más rápido que la luz, porque de hacerlo habrı́a sistemas de referencia
perfectamente válidos donde dicho objeto llegarı́a a su destino antes de haber salido. Esta
es la razón principal por la que, de acuerdo a la relatividad, nada puede viajar más rápido
que la luz: destruirı́a las relaciones de causalidad. La Figura 1.5 muestra las posibles
relaciones causales entre distintos eventos.

1.7. Invariancia del intervalo y el espacio–tiempo de


Minkowski
La relatividad especial, que inició con los trabajos de Einstein de 1905, era una nueva
cinemática basada en los dos postulados de Einstein y en las transformaciones de Lorentz.
Sin embargo, al estar formulada en términos puramente algebraicos, esta teorı́a resultaba
poco intuitiva y sus consecuencias fı́sicas eran difı́ciles de interpretar. Esto cambió en
1908 cuando Hermann Minkowski mostró que la relatividad especial podı́a reinterpretarse
como una teorı́a geométrica en un espacio–tiempo de 4 dimensiones con una medida de
20 CAPÍTULO 1. RELATIVIDAD ESPECIAL

A
C

Figura 1.5: Posibles relaciones causales entre eventos. El evento A y el evento B pueden tener
una relación causal pues están dentro de sus respectivos conos de luz. Lo mismo el evento B y
el evento C. Pero el evento A y el evento C no pueden tener relación casual al estar fuera de sus
conos de luz.

“distancia” invariante [83].16 Inicialmente Einstein vio este nuevo desarrollo como algo
abstracto e innecesario, pero poco a poco se convenció de que en realidad esta nueva
visión era fundamental, y resultó crucial para el posterior desarrollo de la relatividad
general.17
Para entender el resultado de Minkowski partimos de la siguiente definición: dados dos
eventos A y B con coordenadas (tA , xA , yA , zA ) y (tB , xB , yB , zB ), definimos el intervalo
de Minkowski como:
s2AB := t 2 + x2 + y 2 + z 2 , (1.7.1)
donde tomamos t = tA tB , y lo mismo para las otras coordenadas. Nótese que esta
definición es muy similar a la distancia euclidiana en 4 dimensiones (esencialmente el
teorema de Pitágoras), salvo por el signo del término t2 . Definido ası́, el intervalo entre
dos eventos distintos puede ser positivo, negativo, o incluso cero, por lo que s2 no es
16
Minkowski mostró sus resultados originalmente en una conferencia durante la reunión anual de
“Filósofos Naturales” (Naturforscher) en la ciudad de Colonia, Alemania, en septiembre de 1908. Su frase
inicial en dicha conferencia se ha vuelto famosa: “La noción de espacio y tiempo que deseo presentarles
proviene del campo de la fı́sica experimental. Es de ahı́ de donde obtiene su fortaleza. Su tendencia es
radical. Desde ahora, el espacio en sı́ mismo, y el tiempo en sı́ mismo, están destinados a convertirse en
meras sombras, y solo una especie de unión entre ellos mantendrá una realidad independiente”.
17
Einstein declaró que “desde que los matemáticos invadieron la teorı́a de la relatividad, ya ni yo
mismo la entiendo”.
1.7. INVARIANCIA DEL INTERVALO 21

necesariamente el cuadrado de un número real y se debe entender más bien como una
forma cuadrática (cuando necesitemos sacar la raı́z tomaremos el valor absoluto).
Si calculamos ahora el valor del intervalo en el sistema Ō, y usamos las transformaciones
de Lorentz, obtenemos:

s̄2 = t̄2 + x̄2 + ȳ 2 + z̄ 2


= 2
( tv x)2 + 2 ( x v t)2 + y 2 + z2
2
⇥ ⇤
= 1 v2 t 2 + x2 + y 2 + z 2
= t 2 + x 2 + y 2 + z 2 = s2 . (1.7.2)

Las transformaciones de Lorentz entonces garantizan que el intervalo de Minkowski entre


dos eventos arbitrarios es absoluto, es decir tiene el mismo valor en todos los sistemas
inerciales. El intervalo de Minkowski nos da entonces una noción de distancia invariante
(distancia cuadrada en realidad) en el espacio–tiempo de cuatro dimensiones, por lo que
se le conoce también como la pseudo–distancia.
El hecho de que la pseudo–distancia no sea positiva definida no representa en realidad
un problema. Por el contrario, nos permite clasificar la separación entre eventos. Notemos
primero que si dos eventos son tales que el intervalo entre ellos es s2 = 0, esto no
significa que sean el mismo evento. En ese caso tenemos:
r
t2 + x2 + y2 + z 2 = 0 =) = ±1 , (1.7.3)
t
p
donde hemos definido r := x2 + y 2 + z 2 como la distancia espacial euclidiana
(tridimensional) entre ambos eventos. Vemos entonces que si el intervalo entre dos eventos
es cero, esto significa que es posible llegar de uno al otro moviéndose a la velocidad de la
luz, es decir están conectados por una lı́nea a 45 en un diagrama espacio–tiempo.
De acuerdo al signo del intervalo, clasificamos la separación entre dos eventos de la
siguiente manera:18

s2 > 0: Separación espacialoide. Los eventos están más separados en el espacio


que en el tiempo ( r2 > t2 ).

s2 < 0: Separación temporaloide. Los eventos están más separados en el tiempo


que en el espacio ( t2 > r2 ).
18
Algunas referencias definen el intervalo de Minkowski con el signo opuesto al que estamos usando
aquı́: s2 = t2 x2 y2 z 2 . Debido a eso la definición de separaciones espacialoides y tempo-
raloides también cambia de signo.
22 CAPÍTULO 1. RELATIVIDAD ESPECIAL

B C

D
A

Figura 1.6: Tres tipos de intervalos. El intervalo entre los eventos A y B es temporaloide, entre
A y C es nulo, y entre A y D es espacialoide.

s2 = 0: Separación nula (o luminoide). Los eventos están igualmente separados en


el espacio y en el tiempo ( r2 = t2 ), por lo que pueden conectarse por un haz de
luz.

La Figura 1.6 muestra ejemplos de los tres tipos de intervalos. La clasificación de los
intervalos está ı́ntimamente relacionada con los conos de luz. Si dos eventos están dentro
de sus conos de luz su separación es temporaloide, su están fuera de sus conos de luz su
separación es espacialoide, y si están sobre la frontera de sus conos de luz su separación
es nula. De nuestra discusión sobre la causalidad vemos entonces que dos eventos solo
pueden tener una relación causal si su separación es temporaloide o nula.
Como veremos más adelante, en la relatividad ni las distancias espaciales, ni los inter-
valos de tiempo, son invariantes ante transformaciones de Lorentz. Aún más, el hecho de
que la simultaneidad no sea absoluta implica que ni siquiera podemos definir el “espacio
tridimensional” a un tiempo constante de manera absoluta: cada sistema de referencia
tiene su propia definición de qué es el “espacio”. Ante una transformación de Lorentz el
espacio y el tiempo se mezclan. Pero el intervalo de Minkowski permanece invariante. Es
precisamente por esto que hablamos del espacio–tiempo de cuatro dimensiones como un
concepto unificado, y ya no de espacio y tiempo por separado.
Dado que el intervalo de Minkowski nos permite medir distancias en el espacio–tiempo,
decimos que el espacio–tiempo de la relatividad es un espacio métrico. Es importante re-
cordar que en la fı́sica newtoniana no tenemos una medida de distancia en cuatro dimen-
siones, tenemos intervalos de tiempo (absolutos) por un lado, y distancias tridimensionales
(relativas) por otro.19 La existencia de una distancia (o pseudo–distancia) absoluta en el
19
La distancia tridimensional entre dos eventos no es absoluta en la fı́sica newtoniana. Por ejemplo,
si consideramos dos eventos que ocurren en el mismo lugar, pero a distintos tiempos, en un sistema de
1.7. INVARIANCIA DEL INTERVALO 23

espacio–tiempo es sin duda el resultado más fundamental de la relatividad especial. De


hecho, a partir de ahora podemos olvidarnos de los postulados de Einstein y simplemente
definir a la relatividad especial como aquella teorı́a basada en postular que el espacio–
tiempo es un continuo de cuatro dimensiones con una medida de distancia invariante dada
por el intervalo de Minkowski, y una estructura causal dada por los conos de luz. Todo lo
demás se puede obtener a partir de aquı́.

Antes de terminar esta sección vamos a introducir un poco más de notación. Primero,
definimos la siguiente matriz 4 ⇥ 4:
0 1
1 0 0 0
B 0 1 0 0 C
⌘ := B
@ 0 0 1 0 A.
C (1.7.4)
0 0 0 1

A esta matriz se le conoce como el tensor de Minkowski (definiremos el concepto de tensor


más adelante). En términos de sus componentes tenemos:
8
< 1 ↵= =0 ,
⌘↵ = +1 ↵ = 6= 0 , (1.7.5)
:
0 ↵ 6= .
Nótese que para referirnos a las componentes del tensor de Minkowski hemos usado dos
subı́ndices, y no un subı́ndice y un superı́ndice como en el caso de la matriz jacobiana
de la transformación de Lorentz. Veremos más adelante a que se debe esto. Usando la
definición del tensor de Minkowski, el intervalo se puede reescribir ahora como:
3 X
X 3
2
s = ⌘↵ x ↵ x = ⌘↵ x↵ x , (1.7.6)
↵=0 =0

donde en la segunda igualdad hemos usado de nuevo la convención de la suma de Einstein


para simplificar la expresión, solo que ahora hay dos sumas, una sobre ↵ y otra sobre
. En general vamos a utilizar el intervalo en forma diferencial para eventos separados
infinitesimalmente, por lo que tendremos:

ds2 = ⌘↵ dx↵ dx . (1.7.7)


referencia O, es decir que tienen la misma coordenada x, la distancia entre ellos será cero. Pero en un
sistema de referencia Ō que se mueve respecto a O los eventos tendrán distintos valores de x̄, por lo que
la distancia entre ellos será distinta de cero. Nótese que no estamos hablando de la longitud de un objeto
fı́sico, sino de la distancia entre dos eventos. Regresaremos a la noción de longitud más adelante.
24 CAPÍTULO 1. RELATIVIDAD ESPECIAL

A esta última expresión se le conoce como el elemento de lı́nea, pues nos permite calcular
distancias infinitesimales e integrarlas para calcular la longitud de una curva arbitraria
(veremos ejemplos de esto más adelante).

1.8. Parámetro de velocidad e hipérbolas invariantes


Las transformaciones de Lorentz se pueden interpretar de una manera gráfica en un
diagrama espacio–tiempo como un tipo de “rotación”. Para ver esto partimos de definir
el parámetro de velocidad como:

:= arctanh(v) =) v = tanh( ) . (1.8.1)

Nótese que en este definición usamos la tangente hiperbólica y no la esférica. A partir de


la definición anterior es fácil mostrar que:

cosh( ) = , sinh( ) = v . (1.8.2)

En términos del parámetro de velocidad el jacobiano de la transformación de Lorentz


toma entonces la forma (por simplicidad consideramos solo las componentes t y x):
✓ ◆
cosh( ) sinh( )
⇤ := . (1.8.3)
sinh( ) cosh( )

La matriz anterior tiene una estructura que recuerda a la de las rotaciones rı́gidas en el
plano. En efecto, si consideramos el plano euclidiano (x, y), una rotación rı́gida por un
ángulo está dada por la matriz:
✓ ◆
cos( ) sin( )
R := . (1.8.4)
sin( ) cos( )

Esta matriz tiene la misma estructura que la transformación de Lorentz, salvo por un
signo, y por el hecho de que en la transformación de Lorentz aparecen funciones trigo-
nométricas hiperbólicas. El signo distinto se debe simplemente a que en el caso de las
transformaciones de Lorentz los ejes t y x rotan en direcciones opuestas. Por otro lado,
el hecho de que aparezcan funciones hiperbólicas nos indica que la rotación es sobre las
hipérbolas dadas por x2 t2 = ±a2 (con a constante), y no sobre los cı́rculos x2 + y 2 = a2
como en el caso del plano euclidiano.
Las hipérbolas x2 t2 = ±a2 se conocen como las hipérbolas invariantes, debido a
que el intervalo invariante entre un evento arbitrario (t, x) y el origen es precisamente
1.9. TIEMPO PROPIO Y DILATACIÓN DEL TIEMPO 25

s2 = x2 t2 . De la misma manera que un cı́rculo x2 + y 2 = a2 es el lugar geométrico


de todos los puntos que son equidistantes al origen en el plano euclidiano, en el caso del
espacio–tiempo las hipérbolas invariantes x2 t2 = ±a2 marcan el lugar geométrico de
todos los eventos que son equidistantes al origen usando la pseudo–distancia de Minkowski.
Estas hipérbolas tienen como ası́ntotas las lı́neas a 45 en el diagrama espacio–tiempo,
que corresponden al cono de luz del origen, y que pueden interpretarse como hipérbolas
degeneradas (aquellas que están a una pseudo–distancia nula del origen).

Las hipérbolas invariantes nos permiten calibrar los ejes coordenados ante una transfor-
mación de Lorentz simplemente arrastrando puntos sobre la hipérbola correspondiente.20
Por ejemplo, ante una transformación de Lorentz inversa ( ! ), el punto (t̄ = 0, x̄ = 1)
que se encuentra a una distancia unitaria espacialoide del origen s̄2 = 1, corresponde al
punto (t = sinh( ), x = cosh( )) sobre la hipérbola unitaria x2 t2 = 1, que también se
encuentra a una distancia unitaria del origen s2 = sinh2 (v) + cosh2 (v) = 1. De la mis-
ma manera, el punto (t̄ = 1, x̄ = 0) que se encuentra a una distancia unitaria temporaloide
del origen s̄2 = 1, corresponde al punto (t = cosh( ), x = sinh( )) sobre la hipérbo-
la unitaria x2 t2 = 1, que también se encuentra a una distancia unitaria del origen
s2 = cosh2 (v) + sinh2 (v) = 1. Estos conceptos están ilustrados en la Figura 1.7.

1.9. Tiempo propio y dilatación del tiempo


Una de las consecuencias más importantes de la relatividad está relacionada con la
manera en la que fluye el tiempo en los distintos sistemas de referencia. A diferencia de
lo que ocurre en la fı́sica newtoniana donde el tiempo es absoluto, en la relatividad el
tiempo fluye a distintos ritmos en los distintos sistemas de referencia inerciales, es decir
el flujo del tiempo es relativo.
Para entender esto vamos a partir de una definición: definimos el tiempo propio entre
dos eventos infinitesimalmente cercanos d⌧ como el tiempo medido por un reloj ideal
que pasa por ambos eventos.21 Nótese que el tiempo propio solo se puede definir para
eventos con una separación temporaloide, pues en el caso de estar separados de manera
20
Nótese que ante una rotación hiperbólica, el ángulo de rotación de los ejes no es el parámetro de
velocidad = arctanh(v), sino ✓ = arctan(v). El parámetro no es en realidad un ángulo, sino que más
bien nos dice que tanto hay que arrastrar los puntos a lo largo de la hipérbola.
21
Por reloj ideal entendemos un reloj cuyo funcionamiento no se ve afectado por su estado de movi-
miento. Un reloj de arena o de péndulo están muy lejos de ser ideales, pues incluso pequeñas aceleraciones
afectan su funcionamiento severamente. Un reloj de pulsera moderno, por otro lado, funciona muy bien
mientras no consideremos aceleraciones enormes, y es una buena aproximación a un reloj ideal en situa-
ciones prácticas.
26 CAPÍTULO 1. RELATIVIDAD ESPECIAL
_
t t
q

_
B x
q

Figura 1.7: Hipérbolas invariantes y calibración de los ejes. El evento A tiene coordenadas
(t̄ = 1, x̄ = 0) y (t = cosh( ), x = sinh( )), mientras que el evento B tiene coordenadas
(t̄ = 0, x̄ = 1) y (t = sinh( ), x = cosh( )). Ambos están sobre hipérbolas invariantes unitarias.

espacialoide el reloj tendrı́a que moverse más rápido que la luz para pasar por ambos
eventos. Consideremos ahora un sistema inercial Ō que se mueve con el reloj, de manera
que el reloj está en reposo respecto a ese sistema. Dado que el reloj está en reposo en Ō,
tenemos evidentemente que dx̄i = 0, por lo que el intervalo invariante se reduce a:

ds2 = dt̄2 ⌘ d⌧ 2 , (1.9.1)

de donde encontramos:
1/2
d⌧ 2 = ds2 =) d⌧ = ds2 . (1.9.2)

Es decir, el tiempo propio entre dos eventos con separación temporaloide no es más que
la raı́z cuadrada de (menos) el intervalo de Minkowski. Para separaciones espacialoides
tenemos ds2 > 0, por lo que el tiempo propio no tiene sentido pues resultarı́a ser imagi-
nario. Por otro lado, para separaciones nulas el tiempo propio resulta ser igual a cero, lo
que indica que para la luz el tiempo no transcurre.
Dado que hemos definido el tiempo propio de manera infinitesimal, podemos ahora
imaginarnos un reloj que se mueve de manera que su velocidad no sea constante. Como el
intervalo de Minkowski es la medida de distancia invariante en el espacio–tiempo, vemos
que el tiempo propio total en realidad no es otra cosa que la longitud de arco de la lı́nea
1.9. TIEMPO PROPIO Y DILATACIÓN DEL TIEMPO 27

universo de este reloj. Podemos ahora pensar en esa lı́nea universo como una curva en el
espacio–tiempo parametrizada por un parámetro real , de manera que está descrita de la
forma x↵ ( ). El tiempo propio a lo largo de la curva estará dado entonces por la integral:
Z ✓ ◆ Z " 1/2
# Z  1/2
d⌧ ( ds2 ) dx↵ dx
⌧= d = d = ⌘↵ d . (1.9.3)
d d d d

Consideremos ahora un reloj que se mueve en la dirección x con respecto al sistema


del laboratorio O. La invariancia del intervalo implica que:

ds2 = ds̄2 =) dt2 + dx2 = dt̄2 + dx̄2 . (1.9.4)

Pero en el sistema del reloj tenemos dx̄ = 0 y dt = d⌧ (por definición), de manera que:

dt2 + dx2 = d⌧ 2 =) dt2 = dx2 + d⌧ 2 > d⌧ 2 . (1.9.5)

Esto implica que el tiempo medido en el sistema del laboratorio siempre es mayor al
tiempo medido por el reloj en movimiento. En otras palabras, el reloj en movimiento
se atrasa visto desde el sistema del laboratorio. Esto se conoce como la dilatación del
tiempo.22 De hecho, podemos ir más allá. Como el reloj se mueve respecto al sistema del
laboratorio con velocidad v, tenemos dx = vdt, y sustituyendo en la expresión anterior
encontramos:
1
1 v 2 dt2 = d⌧ 2 =) dt2 = d⌧ 2 , (1.9.6)
(1 v 2 )
y finalmente:
dt = d⌧ , (1.9.7)
p
con = 1/ 1 v 2 el factor de Lorentz. Nótese que el factor de Lorentz es igual a 1 para
v = 0, y crece hasta volverse infinito para v = 1, de manera que para |v| > 0 siempre es
mayor que 1 por lo que dt siempre es mayor (o igual para v = 0) que d⌧ .

Aquı́ es muy importante enfatizar que el efecto de dilatación del tiempo es totalmente
simétrico. Si un reloj en reposo en el sistema Ō se atrasa visto desde el sistema O, un reloj
en reposo en el sistema O también se atrasa (¡no se adelanta!) visto desde el sistema Ō.
¿Cómo puede ser esto? ¿Qué no es contradictorio? La respuesta es que no hay ninguna
22
En su derivación de las transformaciones de coordenadas Lorentz habı́a ya encontrado la transforma-
ción de t y hablaba del “tiempo local”, pero lo entendı́a como un artificio matemático y no consideró la
posibilidad de que fuera el tiempo percibido por un observador en movimiento.
28 CAPÍTULO 1. RELATIVIDAD ESPECIAL

contradicción, y todo tiene que ver con qué es lo que estamos comparando con qué. Cuando
en un sistema de referencia especı́fico O comparamos un reloj en movimiento con relojes
en reposo, lo hacemos usando nuestras propias superficies de simultaneidad. Mientras que
los relojes que se mueven respecto a nosotros hacen las comparaciones usando sus propias
superficies de simultaneidad. Esto es más fácil de entender con un diagrama espacio–
tiempo. La Figura 1.8 muestra la lı́nea universo de dos relojes, uno que se encuentra en
reposo en el sistema O (reloj 1), y otro que se mueve respecto a este a velocidad constante v
(reloj 2). Hemos exagerado los ángulos para hacer más claro el diagrama. Por simplicidad
asumimos que las trayectorias de ambos relojes se cruzan en el origen. Los eventos A (el
origen) y B están sobre la lı́nea universo del reloj 2, mientras que los eventos A, C y D se
encuentran sobre la lı́nea universo del reloj 1. El evento C es simultáneo con B de acuerdo
al sistema de referencia O del reloj 1, mientras que el evento D es simultáneo con B de
acuerdo al sistema de referencia Ō del reloj 2.
En el sistema de referencia O comparamos el tiempo propio del reloj en reposo (reloj
1), con el tiempo propio del reloj en movimiento (reloj 2), utilizando las superficies de
simultaneidad de O, y concluimos que ⌧AC > ⌧AB , por lo que el reloj en movimiento
respecto a O se atrasó. Por otro lado, en el sistema Ō se debe comparar el tiempo propio
del reloj en reposo en ese sistema (reloj 2), con el tiempo propio del reloj que se mueve
respecto a ese sistema (reloj 1), usando las superficies de simultaneidad de Ō, y concluimos
que ⌧AB > ⌧AD , por lo que de nuevo el reloj que se mueve respecto a Ō se atrasó. Es
decir, ambos sistemas ven que el reloj que se mueve respecto a ellos es el que se atrasa,
pero están haciendo comparaciones utilizando superficies de simultaneidad distintas. En
realidad no hay ninguna contradicción, pues lo que hemos encontrado es simplemente
que ⌧AC > ⌧AB > ⌧AD , lo que puede verificarse fácilmente calculando los intervalos
invariantes correspondientes.
Aquı́ hay que tener mucho cuidado. Al hacer diagramas espacio–tiempo y comparar in-
tervalos invariantes es muy importante no dejarse guiar por las distancias euclidianas. Los
eventos (A, B, C) forman un triángulo rectángulo, y en términos de distancias euclidianas
vemos claramente que dAC < dAB (el cateto es más corto que la hipotenusa). Pero en el
espacio–tiempo no debemos usar distancias euclidianas sino el intervalo de Minkowski, y
en términos de ese intervalo invariante encontramos ⌧AC > ⌧AB .

La dilatación del tiempo nos lleva directamente a un resultado sorprendente. Con-


sideremos ahora dos relojes en movimiento, el reloj 1 se mueve a velocidad constante,
mientras que el reloj 2 se acelera y desacelera de manera que las trayectorias se cruzan en
dos eventos A y B (ver Figura 1.9). Dado que el reloj 1 se mueve a velocidad constante,
existe un sistema de referencia inercial donde siempre está en reposo. En cambio, dado
que el reloj 2 se acelera no está siempre en reposo en ningún sistema inercial. Sea ahora
1.9. TIEMPO PROPIO Y DILATACIÓN DEL TIEMPO 29

t _
t
Trajectoria del reloj 2
Trajectoria del reloj 1

_
x
C
D B

A
x
Figura 1.8: Lı́nea universo de dos relojes. El reloj 1 está en reposo en el sistema O, mientras que
el reloj 2 está en reposo en el sistema Ō que se mueve respecto a O. Ambos sistemas ven que el
reloj que se mueve respecto a ellos se atrasa, pero las comparaciones se hacen usando distintas
superficies de simultaneidad por lo que no hay contradicción.

Ō el sistema inercial en reposo del reloj 1. En este sistema el reloj 2 siempre está en
movimiento, primero se aleja del reloj 1 y luego se acerca de nuevo (por supuesto hay un
momento intermedio donde el reloj 2 está momentáneamente en reposo respecto al reloj 1,
pero esto ocurre solo en un instante de tiempo). Supongamos que en el primer encuentro
los dos relojes marcan la misma hora. Dado que el reloj 2 siempre se mueve respecto al
reloj 1, visto desde el sistema Ō se atrasa durante todo el trayecto, por lo que cuando se
encuentran por segunda ocasión mide un tiempo menor. Es decir:
Z B Z B
d⌧1 > d⌧2 . (1.9.8)
A A

En esta ocasión la situación ya no es simétrica, y no interviene para nada la definición


de simultaneidad pues estamos comparando los relojes cuando coinciden en los mismos
eventos. Lo que ocurre es que estamos comparando el tiempo propio a lo largo de dos
trayectorias distintas que coinciden en sus extremos. Una de esas trayectorias es una
lı́nea recta en el espacio–tiempo (la del reloj 1 que se mueve a velocidad constante), y
la otra es una curva en el espacio–tiempo (la del reloj 2 que se acelera y desacelera). Y
de la misma manera que un espacio euclidiano podemos distinguir lı́neas rectas de lı́neas
curvas, en el espacio–tiempo de Minkowski también las podemos distinguir. El resultado
es que el reloj que siguió la trayectoria curva se atrasa. Recordemos que para trayectorias
temporaloides el tiempo propio mide directamente la longitud de la curva en términos de
30 CAPÍTULO 1. RELATIVIDAD ESPECIAL

t
B
Reloj 2
Reloj 1

x
Figura 1.9: Dos relojes siguen trayectorias distintas para llegar del evento A al evento B. El
reloj 1 se mueve a velocidad constante, mientras que el reloj 2 acelera y desacelera. El reloj 2
mide un tiempo menor entre los dos eventos.

la pseudo–distancia de Minkowski. Lo que acabamos de demostrar es que la longitud de


arco integrada a lo largo de ambas trayectorias no es la misma.
Esto en realidad no deberı́a sorprendernos, en el espacio euclidiano la longitud de una
lı́nea recta y de una curva que coinciden en dos puntos también resulta ser distinta. Lo
sorprendente en este caso es que, mientras que en el espacio euclidiano una lı́nea recta es
la que tiene la longitud mı́nima entre dos puntos, en el espacio–tiempo de Minkowski una
lı́nea recta temporaloide resulta tener longitud máxima (es la de mayor tiempo propio).23
Podemos utilizar el resultado anterior para reescribir la primera ley de Newton en un
lenguaje geométrico: objetos libres de fuerzas externas se mueven en trayectorias tempora-
loides que maximizan el tiempo propio ⌧ . Estas trayectorias extremas son precisamente
las lı́neas rectas temporaloides. En general, a las curvas de longitud extrema en un es-
pacio dado se les conoce como geodésicas, por lo que la primera ley de Newton equivale
a decir que los objetos libres de fuerzas externas se mueven en geodésicas temporaloides
del espacio–tiempo.24 Por su parte, la luz (y de hecho cualquier partı́cula de masa cero
como veremos más adelante) se mueve a lo largo de geodésicas nulas del espacio–tiempo.
Resulta que esta manera de escribir la primera ley de Newton en términos de geodésicas
se puede generalizar de manera inmediata al caso de un espacio–tiempo curvo como los
23
Para curvas espacialoides en el espacio–tiempo la lı́nea recta tiene longitud mı́nima igual que en un
espacio euclidiano, mientras que una lı́nea recta nula resulta ser un punto silla. En todo caso, las lı́neas
rectas en el espacio–tiempo de Minkowski siempre tienen longitud extrema.
24
La palabra “geodésica” tiene su origen en la cartografı́a terrestre, donde se referı́a a las trayectorias
más cortas entre dos puntos de la Tierra sobre la superficie curva del planeta.
1.10. EL ASUNTO DE LOS GEMELOS 31

que veremos más adelante al estudiar la relatividad general.

1.10. El asunto de los gemelos


El resultado que acabamos de mostrar en la sección anterior es el origen de la famosa
“paradoja” de los gemelos. Aunque como veremos en realidad no hay ninguna paradoja,
por lo que preferiremos referirnos a este ejemplo como el asunto de los gemelos.
La idea es la siguiente. Tenemos dos gemelos idénticos de 20 años de edad. El gemelo 1
se queda en la Tierra, y el gemelo 2 viaja a alta velocidad a una estrella lejana y regresa.
Para ser concretos, asumamos que el gemelo 2 viaja a la estrella Sirio que se encuentra a
8.6 años luz de distancia, y le toma 10 años llegar visto desde la Tierra y otros 10 años
volver, por lo que viaja a una velocidad delp86 % de la velocidad de la luz (v = 0.86). A
esa velocidad el factor de Lorentz es = 1/ 1 0.862 = 1.96 ⇠ 2.
Desde el punto de vista del gemelo 1 han transcurrido 20 años para cuando su hermano
regresa, y ahora tiene 40 años de edad. Sin embargo, debido a la dilatación del tiempo, el
gemelo 2 solo ha envejecido 10 años (el factor de Lorentz es igual a 2), por lo que al volver
tiene solo 30 años de edad. Este es el resultado correcto y no tiene nada de paradójico. El
gemelo 1 se quedó en reposo en un sistema inercial, el de la Tierra (podemos ignorar el
movimiento de la Tierra alrededor del Sol que de cualquier manera es muy lento comparado
con la velocidad de la luz), por lo que siguió una lı́nea recta en el espacio–tiempo, es decir
una geodésica. El gemelo 2, por el contrario, no siguió una trayectoria recta. Incluso si
asumimos que iba a velocidad constante en el viaje de ida y vuelta, su trayectoria en el
espacio–tiempo está formada por dos lı́neas rectas distintas. Las trayectorias de ambos
gemelos forman un triángulo en el espacio–tiempo (ver Figura 1.10), y es claro que incluso
en un espacio euclidiano la suma de dos lados de un triángulo nunca es igual al tercer lado.
Lo interesante, como ya hemos mencionado, es que debido a que se trata de trayectorias
temporaloides en el espacio–tiempo de Minkowski, el lado del triangulo que corresponde
al gemelo 1 que se quedó en la Tierra resulta tener longitud mayor a la suma de los otros
dos lados que corresponden a la trayectoria del gemelo 2. De hecho, si la velocidad del
gemelo 2 se aproxima a la de la luz, el tiempo propio a lo largo de su trayectoria se acerca
a cero. Si pudiera alcanzar la velocidad de la luz su trayectoria habrı́a sido a lo largo
de lı́neas rectas nulas, por lo que no habrı́a envejecido en absoluto (pero veremos más
adelante que no es posible que un objeto con masa alcance la velocidad de la luz).
¿Por qué el resultado puede sonar paradójico? Porque si asumimos que el gemelo 2
va a velocidad constante en cada parte de su trayecto, desde su punto de vista durante
ambos segmentos los relojes en la Tierra deberı́an ser los que se habrı́an atrasado. ¿Cómo
es posible entonces que al llegar de vuelta a la Tierra resulte que no fue ası́? La res-
32 CAPÍTULO 1. RELATIVIDAD ESPECIAL

t
C

E
B
Gemelo 1 Gemelo 2
D

A
Tierra Sirio x
Figura 1.10: Paradoja de los gemelos. El gemelo 1 se queda en la Tierra y el gemelo 2 hace un
viaje de ida y vuelta a Sirio.

puesta es que el gemelo 2 no estuvo siempre en un mismo sistema inercial, sino en dos
sistemas inerciales distintos, por lo que en cada segmento está utilizando superficies de
simultaneidad diferentes para hacer la comparación.
En la Figura 1.10 hemos marcado una serie de eventos de interés. Sea O el sistema de
referencia inercial del gemelo 1. El evento A es el inicio del viaje, el evento C el final, y
el evento B es cuando el gemelo 2 llega a Sirio y da la vuelta. Pero hay otros dos eventos
de interés. El evento D es simultáneo con el evento B en el sistema inercial Ō del gemelo
2 mientras viaja a Sirio, mientras que el evento E es simultáneo con B en el sistema
¯ del gemelo 2 a su regreso a la Tierra. ¿Qué mide el gemelo 2
de referencia inercial Ō
en sus sistemas inerciales? Al llegar a Sirio, y antes de dar la vuelta, compara su reloj
con un reloj en la Tierra a lo largo de su superficie de simultaneidad y encuentra que
⌧AB > ⌧AD , por lo que concluye que los relojes en la Tierra se atrasan. En el camino
de regreso compara su reloj con el reloj en la Tierra a lo largo de sus nuevas superficies
de simultaneidad y encuentra ⌧BC > ⌧EC , y de nuevo concluye que los relojes en la
Tierra se atrasan. Al final del viaje entonces puede concluir que:
⌧2 ⌘ ⌧AB + ⌧BC > ⌧AD + ⌧EC . (1.10.1)
Pero crucialmente ⌧1 ⌘ ⌧AC 6= ⌧AD + ⌧EC . ¡Falta tomar en cuenta el segmento
⌧DE ! La relación completa entre los distintos segmentos es entonces:
⌧1 ⌘ ⌧AC > ⌧2 ⌘ ⌧AB + ⌧BC > ⌧AD + ⌧EC , (1.10.2)
y aquı́ no hay ninguna contradicción.
1.10. EL ASUNTO DE LOS GEMELOS 33

¿Cómo es posible que el gemelo 2 perdiera de vista todo el segmento ⌧DE ? No es que
lo haya perdido de vista, es más bien que hizo las mediciones en dos sistemas inerciales muy
distintos con sus respectivas definiciones de simultaneidad. Recordemos, la simultaneidad
es una convención.
Una pregunta quizá más interesante serı́a qué ve el gemelo 2, y no qué mide. Medimos
con nuestras reglas rı́gidas y relojes sincronizados en sistemas inerciales, mientras que
vemos con luz que se propaga por el espacio–tiempo. Esta pregunta es más compleja.
Mientras el gemelo 2 se aleja de la Tierra los rayos de luz que emite el gemelo 1 tardan
más y más en alcanzarlo. Para cuando el gemelo 2 llega a Sirio, 5 años después de acuerdo
a su propio reloj y 10 años después en la Tierra, apenas esta recibiendo la señal que el
gemelo 1 envió 2.4 años después de iniciado el viaje (Sirio está a 8.6 años luz). Entonces
claramente “ve” que el gemelo 1 solo ha envejecido 2.4 años. Pero al regresar todo se
invierte, y alcanza cada vez más y más rápido los rayos de luz que vienen desde la Tierra,
de manera que en los 5 años que le toma volver ve a su gemelo envejecer aceleradamente
otros 17.6 años, para un total de 20 años contra los 10 que él ha envejecido. Cuidado, ver
y medir son cosas distintas, ver es un fenómeno óptico, medir es simple cinemática.
Pero en toda esta discusión quizá estamos perdiendo de vista lo más importante.
Al definir un sistema inercial utilizamos relojes sincronizados en todo el espacio para
tener una noción de simultaneidad. Pero está claro que no tenemos relojes sincronizados
moviéndose continuamente al 86 % de la velocidad de la luz de aquı́ a Sirio, y en ambas
direcciones, distribuidos a lo largo de todo el trayecto. Dada esta situación, el hablar de
superficies de simultaneidad para el gemelo 2, aunque perfectamente consistente, resulta
ser algo meramente académico y no particularmente útil. Cuando consideramos distancias
astronómicas la noción de simultaneidad en realidad deja de tener mucha utilidad. Lo
importante es que ambos gemelos partieron de un mismo evento, siguieron trayectorias
distintas en el espacio–tiempo hasta volver a encontrarse, y como consecuencia el tiempo
propio total que experimentaron es distinto.

Aquı́ vale la pena eliminar otra duda que surge con frecuencia. Es común encontrar un
argumento que afirma que la asimetrı́a entre ambos gemelos se debe a que uno se acelera
y el otro no. Esto no es correcto. La aceleración en realidad no interviene en el fenómeno,
salvo en el hecho trivial de que el gemelo 2 cambio de dirección, por lo que evidentemente
se aceleró. Lo crucial, de nuevo, es que siguieron trayectorias distintas. Esto se puede ver
fácilmente si consideramos una pequeña variación en el problema. Asumimos ahora que
el gemelo 1 también se acelera en la misma dirección que el gemelo 2, solo que inicia el
viaje tiempo después, se arrepiente rápidamente, y regresa a la Tierra mucho antes que
su hermano. Las trayectorias de ambos gemelos en el espacio–tiempo se pueden ver en la
Figura 1.11. En esta variación del problema ambos gemelos experimentan exactamente las
34 CAPÍTULO 1. RELATIVIDAD ESPECIAL

Gemelo 2
Gemelo 1

Figura 1.11: Una variación en la trayectoria del gemelo 1. Ahora ambos gemelos experimentan
exactamente las mismas aceleraciones, pero aún ası́ el gemelo 2 experimenta un tiempo propio
mucho menor.

mismas aceleraciones aunque en diferentes momentos. Aún ası́, es fácil convencerse que el
gemelo 2 experimenta un tiempo propio mucho menor que el gemelo 1. De hecho, podemos
hacer que el gemelo 1 se arrepienta tan pronto que su tiempo propio sea prácticamente el
mismo que en la versión original del problema (solo ligeramente menor).25

¿Es la dilatación del tiempo un fenómeno real? Sı́, y ha sido verificado experimental-
mente. Aún cuando no tenemos la tecnologı́a para enviar astronautas a Sirio al 86 % de la
velocidad de la luz, sı́ podemos acelerar partı́culas elementales a velocidades mucho más
cercanas a c. Muchas de estas partı́culas son inestables, y decaen después de un cierto
tiempo. Por ejemplo, un neutrón libre en reposo tiene una vida media de unos 880 se-
gundos (poco menos de 15 minutos), mientras que un muón tiene una vida media de solo
2.2 ⇥ 10 6 segundos.26 Cuando las partı́culas se aceleran a velocidades cercanas a la de la
luz estas vidas medias crecen, sus “relojes internos” se atrasan, de manera completamente
consistente con la predicción de la relatividad. Los muones, por ejemplo, fueron descu-
biertos en 1936 estudiando los rayos cósmicos. Dado que su vida media es muy corta en
realidad no nos llegan del espacio sino que se producen en las colisiones de protones (que
sı́ vienen del espacio) con las moléculas de la atmósfera terrestre, a decenas de kilómetros
25
Este argumento está tomado directamente de Maudlin [80].
26
Un muón es una versión inestable y mucho más masiva del electrón, y decae precisamente en un
electrón, un neutrino y un antineutrino.
1.11. CONTRACCIÓN DE LORENTZ–FITZGERALD 35

de altura. No es difı́cil ver que, incluso si se mueven casi a la velocidad de la luz, les
toma aproximadamente ⇠ 10 4 segundos llegar a la superficie terrestre, unas 50 veces
su vida media. Es decir, no deberı́a llegar ninguno. Pero llegan en grandes cantidades: la
dilatación del tiempo hace que su vida media se alargue.

1.11. Contracción de Lorentz–Fitzgerald


Ası́ como la relatividad predice que el flujo del tiempo no es el mismo en todos los
sistemas de referencia, resulta que el tamaño de los objetos tampoco es el mismo en todo
sistema de referencia. En efecto, objetos que se mueven respecto a un cierto sistema de
referencia inercial se contraen en la dirección de movimiento. Esto fenómeno se conoce
como la contracción de Lorentz–Fitzgerald, debido a que Hendrik Lorentz y George Fitz-
gerald lo propusieron (de manera independiente) para explicar los resultados negativos de
los experimentos de Michelson y Morley. Aunque Lorentz y Fitzgerald consideraban que
esta contracción era un efecto dinámico causado por el movimiento del objeto a través al
éter, en la relatividad resulta ser simplemente un efecto cinemático consecuencia de que
la simultaneidad no es absoluta.
Para derivar este resultado consideremos que tenemos un objeto extendido, una barra
por ejemplo, que se mueve con respecto al sistema O con velocidad v > 0. Como es usual,
tomamos Ō como el sistema de referencia en que la barra está en reposo, y denotamos por
L a la longitud de la barra en dicho sistema. Decimos entonces que L es la longitud propia
de la barra. Ahora, desde el punto de vista del sistema O la barra está en movimiento,
por lo que para medir su longitud debemos encontrar la posición de sus extremos a un
mismo tiempo, es decir sobre las superficies de simultaneidad de O. La razón por la que
debemos localizar los extremos a lo largo de una superficie de simultaneidad es que, si la
barra se mueve, no tiene ningún sentido localizar uno de los extremos a un cierto tiempo
y el otro extremo a un tiempo posterior, pues la barra se habrá movido mientras tanto
y eso no nos permitirá medir la longitud correcta. Sea entonces l la longitud de la barra
medida a lo largo de una superficie de simultaneidad O.
La Figura 1.12 muestra la situación que acabamos de describir. El panel b) de la Figura
muestra un acercamiento a la zona de interés. Por simplicidad tomemos al evento A como
el origen. Los eventos A y B corresponden a los extremos de la barra sobre una superficie
de simultaneidad del sistema O, por lo que la longitud de la barra vista en ese sistema es
claramente l = xB . Por otro lado, los eventos A y C corresponden a los extremos de la
barra en la superficie de simultaneidad del sistema Ō donde la barra está en reposo, por
lo que la longitud propia de la barra es L = x̄C . El evento D es simplemente la proyección
de C sobre el eje x, de manera que xC = xD .
36 CAPÍTULO 1. RELATIVIDAD ESPECIAL

a) t - b)
t
C
Trayectoria de la barra
L q t
c
- l d
x
L A B D
q
l x

Figura 1.12: Contracción de Lorentz–Fitzgerald. El panel a) muestra el movimiento de la barra


visto desde el sistema O junto con la longitud propia dada por L, y la longitud en el sistema O
dada por l (ambas medidas a lo largo de las superficies de simultaneidad correspondientes). El
panel b) muestra un acercamiento con algunos eventos de interés mencionados en el texto.

¿Cómo relacionamos L con l? Empezamos por notar que dado que el evento C está so-
bre el eje x̄ por construcción, tenemos t̄C = 0 y x̄C = L. Las transformaciones de Lorentz
(inversas) implican entonces que las coordenadas de C en el sistema O son:

xC = L , tC = vL , (1.11.1)
p
con = 1 v 2 el factor de Lorentz. Por otro lado tenemos l = xB = xD d = L d,
donde usamos el hecho de que xD = xC = L.
Necesitamos ahora encontrar el valor de d. De la figura vemos que los eventos BCD
forman un triángulo rectángulo. Notemos que el ángulo ✓ es el ángulo entre la vertical
y una lı́nea paralela al eje t̄, por lo que debe estar dado por ✓ = arctan(v). Tenemos
entonces:27
d/tC = tan ✓ = v =) d = vtC = v 2 L . (1.11.2)
Encontramos entonces para l:

l= L v2L = L 1 v 2 = L/ 2
, (1.11.3)

y finalmente:
l = L/ . (1.11.4)
27
Una duda común es el por qué podemos utilizar trigonometrı́a estándar si estamos en el espacio–
tiempo de Minkowski. La razón es que para definir el ángulo ✓ no usamos intervalos invariantes, sino
simplemente las coordenadas cartesianas (t, x). En particular, en el cálculo que acabamos de hacer nunca
usamos la longitud de la hipotenusa del triángulo BCD, que podrı́a parecer problemática, sino simple-
mente los catetos cuyas longitudes euclidianas corresponden directamente a las coordenadas x y t.
1.11. CONTRACCIÓN DE LORENTZ–FITZGERALD 37

Dado que para v 6= 0 el factor de Lorentz es tal que > 1, vemos que l es siempre
menor que L, es decir la barra se contrae en la dirección de movimiento.28 Esta es la
famosa contracción de Lorentz–Fitzgerald (aunque por simplicidad de ahora en adelante
la llamaremos simplemente la contracción de Lorentz).

¿Es la contracción de Lorentz un efecto real? Es decir, ¿la barra se contrae, o solo se
“ve” contraı́da? La respuesta es que el efecto es real, la barra sı́ se contrae (ver siguiente
sección). La contracción de Lorentz viene directamente de la relatividad de la simultanei-
dad. En ese sentido es un efecto de proyección, ya que al pasar de un sistema de referencia
a otro rotamos los ejes coordenados y ya no estamos de acuerdo en quién es el “espacio”
sobre el cuál debemos medir la longitud del objeto. Pero no es de ninguna manera una
simple “ilusión óptica”.
De hecho, el efecto óptico es muy distinto, y un objeto que se mueve a velocidades
cercanas a la de la luz no se “ve” contraı́do (aunque sı́ lo está), sino que se ve rotado un
ángulo ' = arcsin(v). Para entender este efecto observemos con cuidado la Figura 1.13.
El panel a) muestra un cubo en reposo rotado un ángulo ' (el observador está abajo),
y el panel b) muestra la imagen que ve el observador, donde hemos coloreado el lado B
para mayor claridad. Si el cubo tiene lado L y el ángulo de rotación es ', la proyección
del lado A que ve el observador tiene una longitud de LA = L cos ', y la proyección del
lado B una longitud de LB = L sin '. Nótesepque en general LA + LB > L, por ejemplo si
tomamos ' = 45 encontramos LA + LB = 2L, lo que no es de extrañar pues estamos
viendo el cubo a lo largo de toda la diagonal.
Por otro lado, los paneles c) y d) muestran la situación para un cubo no rotado que se
mueve hacia la derecha con velocidad v > 0. En el panel c) vemos aquellos rayos de luz
que llegan al observador a un mismo tiempo en el momento en que el cubo está frente a
él. Es claro que todos los rayos de luz del lado A alcanzan al observador simultáneamente.
Ahora, si el cubo estuviera en reposo el observador solo podrı́a ver el lado A y no el lado
B ya que el cubo mismo le estorbarı́a, pero dado que el cubo se está moviendo de hecho
sı́ puede ver el lado B: los rayos que salieron un poco antes desde el lado B lo alcanzan
pues el cubo se movió y ya no estorba. En otras palabras, cuando el cubo está justo frente
a él, el observador sı́ alcanza a ver el lado B gracias a la luz que se emitió antes.
¿Qué tan largo se ve cada lado? La luz que sale del extremo más lejano del lado B
debe recorrer una distancia extra igual a L para llegar al observador, y dado que la luz
28
De nuevo hay que tener cuidado con la intuición euclidiana. Si bien es cierto que si la geometrı́a
fuera euclidiana del diagrama verı́amos inmediatamente que l < L, la razón entre ambas distancias no
serı́a la correcta. Podemos usar trigonometrı́a estándar cuando hablamos de coordenadas, pero no cuando
hablamos de longitudes fı́sicas medidas con reglas (o tiempos medidos con relojes). Para eso debemos
usar el intervalo invariante, o de manera equivalente las transformaciones de Lorentz.
38 CAPÍTULO 1. RELATIVIDAD ESPECIAL

a) b)
L
L
B A

A
j
L sin j L cos j
L sin j L cos j

c) d)
v v

B A
A
d Lv L/g

Figura 1.13: El panel a) muestra un cubo en reposo rotado un ángulo ' con el observador en
la parte de abajo, y el panel b) muestra la imagen que ve dicho observador. Los paneles c) y
d) muestran un cubo que no está rotado pero que se mueve con velocidad v > 0. El panel c)
muestra los rayos de luz que llegan al observador a un mismo tiempo, y el panel d) la imagen que
ve. Debido al movimiento del cubo, el observador sı́ alcanza a ver el lado B, y como resultado
el cubo se ve rotado.
1.12. LA BARRA Y EL GRANERO 39

viaja a velocidad 1 (en unidades geométricas) le toma un tiempo T = L recorrer dicha


distancia. Durante ese tiempo el cubo se movió una distancia d = vT = vL, ası́ que el
observador ve el lado B proyectado con una longitud LB = vL (notamos de nuevo que si
v = 0 el observador no alcanza a ver el lado B). Por otro lado, el lado A está contraı́do
debido a la contracción de Lorentz, por lo que tiene una longitud de LA = L/ .
Pero ahora viene lo más interesante. Dado que p v < 1, podemos
p definir un ángulo
' := arcsin(v).29 En ese caso tenemos cos ' = 1 sin2 ' = 1 v 2 = 1/ . Por lo
que encontramos LA = L cos ' y LB = L sin ', es decir el cubo se ve exactamente como
se verı́a un cubo en reposo rotado un ángulo '. Esto se conoce como la rotación de
Penrose–Terrell, y fue descubierta independientemente por Roger Penrose y James Terrell
en 1958-1959 [102, 125].30

Un último comentario antes de terminar esta sección. La contracción de Lorentz


está ı́ntimamente ligada con la dilatación del tiempo. Por ejemplo, cuando discutimos
el asunto de los gemelos mencionamos que para el gemelo 2 que viaja a Sirio al 86 % de la
velocidad de la luz el factor de Lorentz es ⇠ 2, por lo que durante el viaje de la Tierra
a Sirio solo transcurren 5 años en su sistema de referencia. Pero Sirio está a 8.6 años luz
de la Tierra. ¿Cómo pudo recorrer esa distancia en tan solo 5 años? La respuesta es, por
supuesto, la contracción de Lorentz. Sirio está a 8.6 años luz de la Tierra en el sistema de
referencia en el que ambos están en reposo. Pero en el sistema de referencia del gemelo
2 la distancia entre la Tierra y Sirio se contrae a 4.3 años luz, por lo que no es ninguna
sorpresa que pueda llegar hasta allá en solo 5 años. Lo que en un sistema de referencia se
entiende como dilatación del tiempo, en el otro sistema se entiende como contracción de
Lorentz. Por cierto, la visión correcta es que en su propio sistema de referencia el gemelo
2 no se mueve, y es Sirio el que se acerca a él al 86 % de la velocidad de la luz, al mismo
tiempo que la Tierra se aleja a esa misma velocidad.

1.12. La barra y el granero


Vamos ahora a considerar un ejemplo de como funciona la contracción de Lorentz,
conocido comúnmente como la “paradoja” de la barra y el granero. De nuevo, usar la
palabra paradoja para referirnos a este ejemplo no es correcto ya que no existe ninguna
contradicción y el resultado es completamente consistente. Solo aparenta ser paradójico
si uno no lo piensa con cuidado. Este ejemplo nos servirá, además, para llegar a algunas
conclusiones importantes.
29
No confundir ' con el parámetro de velocidad = arctanh(v).
30
Penrose en particular mostró que una esfera en movimiento siempre se ve con una proyección circular.
40 CAPÍTULO 1. RELATIVIDAD ESPECIAL

a) Trayectoria b) Trayectoria
Trayectoria -
t del granero
de la barra t de la barra
Trayectoria
B del granero
A A
B

x x-
Figura 1.14: La barra y el granero versión 1: el granero tiene dos puertas. El panel a) muestra
la situación vista desde el sistema de referencia del granero O, y el panel b) muestra la situación
vista desde el sistema de referencia de la barra Ō.

El problema es el siguiente: tenemos un granero y una barra que tienen la misma


longitud propia L, que para ser concretos fijaremos en 10 metros. Alejamos la barra del
granero, y aceleramos la barra a velocidades relativistas en dirección
p al granero. De nuevo,
para ser concretos tomemos la velocidad de la barra igual a v = 3/2 = 0.866, de manera
que el factor de Lorentz = 2. A esa velocidad, la barra está contraı́da y su longitud es
de solo 5 metros, por lo que cabe perfectamente en el granero, y una vez que está adentro
cerramos la puerta. Pero aquı́ viene la aparente paradoja: desde el punto de vista de la
barra es ella quién está en reposo y es el granero el que se acerca con una velocidad
v = 0.866. El granero entonces está contraı́do y solo mide 5 metros, por lo que no hay
manera de que la barra, que mide 10 metros en su propio sistema de referencia, quepa en
él. ¿Qué ocurre entonces? ¿La barra entra al granero o no lo hace? ¿La puerta se puede
cerrar? Para entender esto vamos a considerar dos versiones distintas del problema.

En la primera versión asumimos que el granero tiene dos puertas, una adelante y una
atrás. Inicialmente la puerta trasera está cerrada y la puerta delantera está abierta, y hay
sensores en cada puerta. El sensor en la puerta delantera está programado para cerrar la
puerta en cuanto la barra la cruce por completo, llamemos a esto el evento A. Por otro
lado, el sensor en la puerta trasera está programado para abrir esa puerta en el instante
en que la barra está a punto de chocar con ella, llamaremos a esto el evento B.
La Figura 1.14 muestra un diagrama espacio–tiempo de esta situación. En el panel a)
de la Figura se muestra lo que ocurre desde el punto de vista del sistema de referencia O
donde el granero está en reposo. En el evento A la puerta delantera se cierra y la barra
está completamente contenida dentro del granero. En el evento B, que ocurre después, la
1.12. LA BARRA Y EL GRANERO 41

puerta trasera se abre y la barra puede continuar su camino. Lo importante aquı́ es que
en este sistema de referencia A ocurre antes que B, es decir tA < tB .
El panel b) muestra la misma situación vista desde el punto de vista del sistema Ō
donde la barra está en reposo. En este sistema la barra es más larga que el granero. El
granero se mueve hacia la izquierda y comienza a tragarse a la barra. Cuando la parte
trasera del granero está a punto de chocar con el extremo de la barra la puerta trasera
se abre (evento B). La barra ahora está ensartada en el granero con ambos extremos
salidos, mientras el granero se sigue moviendo a la izquierda. Después de un tiempo la
parte delantera del granero llega al final de la barra y la puerta se puede cerrar (evento
A). Vemos que en este sistema de referencia B ocurre antes que A, es decir t̄B < t̄A .
En ambos sistemas de referencia los eventos A y B ocurren sin ningún problema: una
puerta se cierra cuando la barra ya entró (A), y la otra se abre cuando está a punto de salir
(B). ¡Pero el orden temporal de los eventos A y B se invierte! ¿Es eso un problema? No
lo es siempre y cuando los eventos A y B no puedan tener una relación causal entre ellos,
es decir siempre y cuando su separación sea de tipo espacialoide. En la Figura hemos
marcado también con lı́neas punteadas el cono de luz del evento B en ambos paneles.
Podemos ver claramente que el evento A está fuera del cono de luz de B, por lo que
en efecto la separación entre ambos eventos es espacialoide, y para eventos separados de
manera espacialoide el orden temporal no está bien definido, es cuestión de convención.
Concluimos entonces que no hay ninguna paradoja.

En la segunda versión del problema complicamos un poco las cosas: pedimos que solo
exista la puerta delantera, y sustituimos la puerta trasera por un muro sólido capaz de
detener la barra. El evento B ahora será el momento en que la barra choca con la parte
trasera del granero. La Figura 1.15 muestra un diagrama espacio–tiempo de esta nueva
situación. El panel a) muestra de nuevo el punto de vista del sistema del granero O. En
este sistema las cosas siguen de manera similar al caso anterior. La barra está contraı́da,
entra en el granero y la puerta se cierra en el evento A. Posteriormente la barra choca
con la parte posterior del granero en el evento B y se detiene.
En el panel b) vemos la situación desde el punto de vista del sistema de la barra Ō.
Ahora el evento B ocurre antes, la barra choca con la parte de atrás del granero cuando
su extremo izquierdo aún sigue fuera del granero. ¿Cómo puede cerrarse la puerta? La
respuesta es que, aún cuando en el evento B la parte derecha de la barra ya chocó con el
muro del granero y entonces comienza a ser empujada hacia la izquierda por el granero,
el extremo izquierdo de la barra sigue en reposo pues aún no le ha llegado ninguna señal
que le diga que el choque ya ocurrió y “debe empezar a moverse”. Recordemos que la
velocidad máxima de cualquier interacción fı́sica es la velocidad de la luz. Una onda de
choque se propaga entonces a través de la barra hacia la izquierda, pero lo hace a lo más a
42 CAPÍTULO 1. RELATIVIDAD ESPECIAL

a) b)
Trayectoria -
t del granero t
C
Trayectoria C
B del granero
A Trayectoria A B Trayectoria
de la barra de la barra

x x-
Figura 1.15: La barra y el granero versión 2: el granero tiene una puerta en la parte delantera
y un muro sólido en la parte trasera. El panel a) muestra la situación vista desde el sistema de
referencia del granero O, y el panel b) muestra la situación vista desde el sistema de referencia
de la barra Ō.

la velocidad de la luz (en la práctica seguramente mucho más lento). En la Figura hemos
marcado un tercer evento C, que corresponde a la intersección entre la lı́nea universo del
extremo izquierdo de la barra, y el cono de luz de B. Este es el primer momento en que
el extremo izquierdo de la barra puede recibir el efecto de la colisión con el granero, y
hasta ese momento continuará sin moverse. Pero ahora es claro que, en ambos sistemas
de referencia C ocurre después que A. Esto implica que la puerta se cierra antes de que
el extremo izquierdo de la barra reciba el efecto del choque. Para cuando esto ocurre la
barra ya está comprimida dentro del granero.
¿Qué ocurre después? Eso ya depende de la dinámica del material de la barra, y no es
de gran interés. La barra y granero, que ahora están en reposo en un mismo sistema de
referencia, podrı́an quizá regresar a su longitud propia después de algún proceso elástico,
o la barra podrı́a romperse si no logra soportar la colisión. Lo importante es que la barra
entra al granero y la puerta se cierra, y de nuevo no hay ninguna paradoja. Notemos que
el hecho de que la barra se comprima después de la colisión no es debido a la contracción
de Lorentz, sino al hecho de que la interacción fı́sica entre sus átomos viaja, a lo más, a
la velocidad de la luz. Es algo similar a lo que ocurre en la fila del cine si la persona de
hasta atrás empuja a quién le queda enfrente. No todos se caen a la vez, se van cayendo
uno después de otro debido a que la interacción entre las personas no es instantánea.
Podemos entonces concluir varias cosas a partir del problema de la barra y el gra-
nero. Primero, vemos de nuevo que la relatividad de la simultaneidad implica que el
orden temporal de eventos separados de manera espacialoide no está bien definido, y esto
no representa ninguna contradicción. Por otro lado, de la segunda versión del problema
1.13. COMPOSICIÓN DE VELOCIDADES 43

concluimos que en la relatividad no existen los objetos infinitamente rı́gidos, pues esto
implicarı́a que la interacción entre sus partes es instantánea. En los ejercicios al final del
capı́tulo veremos ejemplos de situaciones donde un objeto no solo se comprime debido a
este fenómeno, sino que incluso se puede doblar.

1.13. Composición de velocidades


En la mecánica de Newton las velocidades resultan ser aditivas debido a las transforma-
ciones de Galileo. En efecto, si tenemos un objeto que se mueve con velocidad u = dx/dt a
lo largo del eje x visto desde un sistema de referencia O, y queremos calcular su velocidad
ū = dx̄/dt̄ en un segundo sistema de referencia Ō que se mueve respecto a O con velocidad
v también a lo largo del eje x, encontramos:

dx̄ d(x vt) dx dt


ū = = = v =u v, (1.13.1)
dt̄ dt dt dt
donde hemos utilizado las transformaciones de Galileo (1.5.1).
En la relatividad especial las velocidades resultan ya no ser aditivas, por lo que en
lugar de hablar de la “suma de velocidades” hablamos de la composición de velocidades.
Para encontrar la regla de composición de velocidades en la relatividad partimos de la
misma manera que en el caso que acabamos de ver, solo que ahora en lugar de usar las
transformaciones de Galileo usamos las transformaciones de Lorentz (1.5.14)-(1.5.15). En
ese caso tenemos:
dx̄ d(x vt) dx vdt dx/dt v
ū = = = = , (1.13.2)
dt̄ d(t vx) dt vdx 1 vdx/dt

y finalmente:
u v
ū = . (1.13.3)
1 uv
Esta es la regla de composición de velocidades en relatividad especial (asumiendo movi-
miento solo en el eje x). Notamos de inmediato que la composición de velocidades ya no
es una simple adición.
Hay dos casos lı́mites de interés que vale la pena mencionar. El primero es el caso
cuando ambas velocidades son pequeñas comparadas con la velocidad de la luz, es decir
|u| ⌧ 1 y |v| ⌧ 1. En ese caso tenemos claramente que |uv| ⌧ 1, y la composición de
velocidades se reduce al caso newtoniano. El otro lı́mite es cuando el objeto que estamos
considerando es un fotón (un haz de luz), en cuyo caso tenemos u = 1. Sustituyendo
44 CAPÍTULO 1. RELATIVIDAD ESPECIAL

en (1.13.3) encontramos:
u v 1 v
ū = = =1. (1.13.4)
1 uv 1 v
Esto no es otra cosa que el segundo postulado de Einstein: la velocidad de la luz es la
misma en todo sistema inercial.

1.14. Comentarios filosóficos


Aunque en este capı́tulo hemos seguido una introducción histórica partiendo de los
postulados de Einstein y las transformaciones de Lorentz es importante mencionar que,
si bien la vı́a histórica es ciertamente interesante y nos permite seguir un camino más
intuitivo, no es la manera más formal de presentar a la teorı́a de la relatividad especial.
El aspecto realmente fundamental de la relatividad especial es que postula que el
espacio–tiempo es un continuo de cuatro dimensiones con una geometrı́a caracterı́stica da-
da por el intervalo o pseudo–distancia invariante de Minkowski. El intervalo de Minkowski
se presenta normalmente usando coordenadas cartesianas, pero es importante enfatizar
que dichas coordenadas son solo una manera de localizar eventos en el espacio–tiempo.
En principio las coordenadas no tienen porqué tener una interpretación fı́sica directa, sino
que son simplemente etiquetas que nos permiten diferenciar un evento de otro.
El espacio–tiempo de Minkowski difiere fundamentalmente del espacio y el tiempo
Newtonianos. En la teorı́a de Newton el espacio y el tiempo son cosas muy distintas. No
se habla de un continuo de cuatro dimensiones, sino de un continuo de una dimensión que
llamamos el “tiempo”, al que en cada punto se le asocia otro continuo de tres dimensiones,
es decir el “espacio” a cada tiempo.31
La geometrı́a del espacio–tiempo de Minkowski tiene una estructura causal dada por
los conos de luz, y esta es su propiedad fundamental. El cono de luz de un evento dado
E divide al espacio–tiempo en tres regiones distintas: 1) el futuro causal de dicho evento,
es decir todos aquellos eventos que pueden ser afectados por E (dado por el cono de
luz futuro); 2) el pasado causal de E, que incluye todos los eventos que pudieron haber
afectado a E (dado por el cono de luz pasado); y 3) todos aquellos eventos fuera del cono
de luz de E que no están ni en su futuro ni en su pasado.
A partir de la geometrı́a de Minkowski también podemos definir las lı́neas rectas
en el espacio–tiempo, ası́ como una noción de paralelismo global (esto se conoce como
una estructura afı́n). Dichas lı́neas rectas resultan tener longitud extrema, es decir son
geodésicas: longitud mı́nima para separaciones espacialoides, máxima para separaciones
31
En el lenguaje de la geometrı́a diferencial el espacio–tiempo newtoniano es lo que se conoce como
un haz fibrado, donde el haz es el tiempo R1 , y la fibra es el espacio de tres dimensiones R3 .
1.14. COMENTARIOS FILOSÓFICOS 45

temporaloides, y tipo punto silla para separaciones nulas. Y dadas esas lı́neas rectas po-
demos enunciar la ley de la inercia o primera ley de Newton en un lenguaje puramente
geométrico: partı́culas con masa libres de fuerzas externas siguen lı́neas rectas tempora-
loides (geodésicas) en el espacio tiempo, y partı́culas de masa cero (fotones) siguen lı́neas
rectas nulas en el espacio–tiempo. Ninguna interacción fı́sica puede propagarse a lo largo
de trayectorias espacialoides. Además, postulamos que el intervalo ds2 de Minkowski es
empı́ricamente accesible: para separaciones espacialoidespnos proporciona directamente
las mediciones de longitud (lo que miden las reglas) dl = ds2 , y para separaciones tem-
poraloides nos proporciona
p directamente lo que mide un reloj que se mueve a lo largo de
esa trayectoria d⌧ = 2
ds .
Lo que acabamos de describir es el contenido esencial de la relatividad especial, que
como vemos puede expresarse sin recurrir en ningún momento a un sistema de coorde-
nadas. Por supuesto, dada la estructura geométrica del espacio–tiempo de Minkowski,
podemos definir un sistema de referencia inercial como aquel cuyas lı́neas coordenadas
son lı́neas rectas ortogonales en el espacio–tiempo. Definiremos el concepto de ortogonali-
dad en detalle más adelante. De momento basta con decir que para las lı́neas coordenadas
asociadas al tiempo tomamos simplemente un conjunto lı́neas rectas temporaloides pa-
ralelas entre sı́, mientras que para las lı́neas coordenadas espaciales tomamos un sistema
de coordenadas cartesiano en las superficies de simultaneidad asociadas a las lı́neas de
tiempo (ya hemos visto como podemos establecer esas superficies de simultaneidad sin-
cronizando relojes). En este sentido un sistema de coordenadas inerciales es especial en el
espacio–tiempo de Minkowski exactamente de la misma manera que las coordenadas car-
tesianas son especiales en el espacio euclidiano. Pero al igual que en el espacio euclidiano
podemos decidir trabajar en coordenadas no cartesianas (esféricas, cilı́ndricas, etc.), en el
espacio–tiempo de Minkowski podemos trabajar en coordenadas no inerciales. Como ya
hemos mencionado, las coordenadas son simplemente etiquetas.
El hecho de que tengamos un concepto de paralelismo global y podamos construir un
sistema de coordenadas ortogonales indica que formalmente el espacio–tiempo de Min-
kowski es un espacio plano. Cuando lleguemos a la relatividad general veremos que la
gravedad se asocia a una curvatura del espacio–tiempo, por lo que en presencia de un
campo gravitacional ya no es posible tener un sistema inercial global, aunque la estruc-
tura de Minkowski se mantiene de manera local.

Es importante mencionar, sin embargo, que esta estructura del espacio–tiempo es un


postulado. Dicho postulado nos ha permitido entender, describir y predecir de manera
consistente una enorme cantidad de fenómenos fı́sicos, que van desde la electrodinámica
de Maxwell hasta las teorı́as cuánticas de campo, y en su generalización a espacios curvos
desde las ondas gravitacionales y los agujeros negros, hasta la cosmologı́a y el origen del
46 CAPÍTULO 1. RELATIVIDAD ESPECIAL

Universo. Pero como todo postulado, el espacio–tiempo de la relatividad puede no ser la


descripción más fundamental de la naturaleza. A escalas mucho más pequeñas que las
de los núcleos atómicos, el espacio–tiempo fı́sico podrı́a dejar de ser un continuo para
convertirse quizá en una red de conexiones causales que obedecen reglas cuánticas. O
bien, podrı́amos encontrar que la estructura causal basada en los conos de luz ya no es
válida en todas las circunstancias (el fenómeno del entrelazamiento cuántico parecerı́a
apuntar en esta dirección). Esta es el área de investigación de la gravedad cuántica, que
busca unificar los principios de la relatividad con los de la mecánica cuántica.32 El tiempo
dirá que tan lejos podemos llevar los postulados de la relatividad, pero hasta ahora han
resultado ser enormemente robustos.

32
Se espera que la relatividad general deje de ser válida en la llamada escala de Planck, donde los
fenómenos cuánticos asociados a la naturaleza del espacio–tiempo ya no puedan ser ignorados. Esta
escala se define a partir de las tres constantes más fundamentales de la naturaleza, la velocidad de la luz
c, la constante p
de la gravitación de Newton G, y la constante de Planck
p ~, y corresponde a distancias del
orden de lP = ~G/c3 ⇠ 10 35 m, y tiempos del orden de tP = ~G/c5 ⇠ 10 43 s.
1.15. EJERCICIOS 47

1.15. Ejercicios
1.1. Unidades geométricas. Conversión de unidades geométricas a unidades del sistema
internacional:

a) Convertir de unidades SI a unidades naturales: la constante de Planck


h = 6.626 ⇥ 10 34 J s, la constante de Newton G = 6.67 ⇥ 10 11 N m2 /kg2 , la
aceleración de la gravedad terrestre g = 9.8 m/s2 , y la velocidad del sonido en
el agua v = 1460 m/s.
b) Convertir de unidades naturales a SI: tiempo t = 5 ⇥ 1020 m; aceleración
a = 2 m 1 ; densidad de energı́a u = 20 kg/m3 ; presión p = 5 ⇥ 1010 kg/m3 .

1.2. Ejes coordenados. Un sistema de referencia Ō se mueve respecto a O con velocidad


v > 0 a lo largo del eje x. Demuestre explı́citamente a partir de la definición de
velocidad y la invariancia de la velocidad de la luz (sin usar las transformaciones de
Lorentz) qué:

a) El eje t̄ del sistema Ō gira en el sentido de las manecillas del reloj un ángulo
✓ = arctan v.
b) El eje x̄ del sistema Ō gira en el sentido inverso el mismo ángulo. Hint: utilice
un rayo de luz que sale del eje t̄ a un tiempo t̄ = T , se refleja en un punto
sobre el eje x̄, y regresa al eje t̄ a un tiempo t̄ = +T . ¿Cómo se ve esto desde
el sistema del laboratorio?

1.3. Orden temporal de eventos I. Considere dos eventos A y B tales que en el sistema
O el evento A ocurre antes que el evento B. Demuestre explı́citamente que:

a) Si los eventos A y B están separados de manera temporaloide, A ocurre antes


que B en todo sistema inercial.
b) Si los eventos A y B están separados de manera espacialoide, existen sistemas
inerciales que se mueven con velocidad v tal que |v| < 1, donde B ocurre antes
que A. Encuentre cuál es la velocidad mı́nima v que debe tener el sistema
Ō para que el orden temporal de los eventos se invierta en términos de las
separaciones tAB y xAB .

1.4. Orden temporal de eventos II. Tres eventos A, B y C ocurren en el orden temporal
ABC vistos en el sistema O. Otro observador Ō ve los eventos en el orden CBA.
¿Es posible que un tercer observador vea los eventos en el orden ACB? Justifique
su respuesta con un diagrama espacio–tiempo.
48 CAPÍTULO 1. RELATIVIDAD ESPECIAL

1.5. Ángulo de emisión. Supongamos que un cohete se mueve en la dirección x con velo-
cidad v respecto a la Tierra, y que emite un haz de luz a un ángulo ✓¯ con respecto
a su dirección de propagación medido en su propio sistema de referencia. Muestre
que el ángulo ✓ del haz de luz con respecto al eje x medido desde la Tierra es tal
que:
v + cos ✓¯
cos ✓ =
1 + v cos ✓¯
1.6. Parámetro de velocidad. Si una partı́cula se mueve con velocidad v definimos el
parámetro de velocidad como: ⌘ arctanh(v).

a) Muestre que la ley relativista de composición de velocidades implica que los


parámetros de velocidad se suman de forma directa.
b) Considere ahora un objeto moviéndose respecto al laboratorio con velocidad v.
Un segundo objeto se mueve respecto al primero con velocidad v, un tercero
se mueve respecto al segundo con velocidad v, y ası́ sucesivamente. Si hay N
objetos, ¿Cual es la velocidad del enésimo respecto al laboratorio? (Hint: utilice
el resultado del inciso anterior.)

1.7. Transformaciones de Lorentz. Partiendo de la definición del parámetro de velocidad


:= arctanh(v) muestre qué:

a) En términos de , las transformaciones de Lorentz toman la forma:

t̄ = t cosh x sinh , x̄ = t sinh + x cosh .

b) Use esto para demostrar la invariancia del intervalo: t 2 + x2 = t̄2 + x̄2

1.8. Barra inclinada I. Un sistema de referencia Ō se mueve respecto a O con velocidad


vx a lo largo del eje x. En Ō una barra paralela al eje x̄ se mueve en la dirección ȳ
con velocidad constante uy . Demuestre que en el sistema de referencia O la barra
esta inclinada respecto al eje x con un ángulo ✓ = arctan( vx uy ), con el factor de
Lorentz.

1.9. Barra inclinada II. Un sistema de referencia Ō se mueve respecto a O con velocidad
v > 0 a lo largo del eje x. Una barra en reposo en el sistema Ō tiene un ángulo ✓¯ en
el plano x̄ ȳ con respecto al eje x̄. ¿Cuál es el ángulo respecto al eje x medido en
el sistema O?
1.15. EJERCICIOS 49

1.10. Barra y agujero. Una barra paralela al eje x de longitud propia igual a 1 m se
mueve en la dirección x con velocidad constante vx en el sistema del laboratorio O.
Al mismo tiempo, una placa delgada paralela al plano xy se mueve en la dirección
z (hacia arriba) con velocidad constante uz , también vista desde el laboratorio. La
placa tiene un agujero circular de 1 m de diámetro centrado en el eje z. Las cosas
se sincronizan de tal manera que el centro de la barra llega al origen x = y = z = 0
al mismo tiempo que la placa alcanza el plano z = 0. Visto desde el laboratorio,
la barra esta contraı́da y cabe en el agujero, de manera que la barra pasa a través
del agujero sin problema y sigue su camino mientras la placa sigue subiendo. Pero
desde el sistema de referencia de la barra Ō, el agujero es el que está contraı́do y la
barra no cabe en él. ¿Qué pasa realmente? Resuelva la aparente paradoja y explique
lo que ocurre visto desde el punto de vista del sistema Ō.

1.11. Barra y granero. Una barra de longitud en reposo l se mueve a una velocidad v
hacia un granero de longitud en reposo L.

a) Dada una velocidad fija v, encontrar cual es la longitud máxima de la barra


l > L que permite que la puerta del granero se cierre.
b) Si tomamos l = L, muestre explı́citamente que los eventos: A la parte trasera
de la barra pasa por la puerta del granero (y la puerta se cierra), y B la parte
delantera de la barra choca con el fondo del granero, están separados de manera
espacialoide para toda 0 < v < 1.

1.12. Guerra espacial. Dos cohetes de igual longitud en reposo pasan uno junto al otro
moviéndose en direcciones opuestas a velocidades relativistas. El cohete A tiene un
cañón en la cola y lo dispara contra el cohete B en el instante en que la nariz de
A pasa justo frente a la cola de B. Como desde el punto de vista de A el cohete B
esta contraı́do, A espera fallar el tiro. Pero desde el punto de vista de B, A es quién
esta contraı́do, y B piensa que si A le dispara cuando la punta de A esta frente a
la cola de B el tiro no puede fallar. ¿Que pasa realmente? Indique que es lo que no
queda totalmente claro en el planteamiento del problema. Diga cuales son los dos
planteamientos posibles que son consistentes y que ocurre en cada caso.

1.13. Composición de velocidades. Consideremos un objeto que se mueve con velocidad


tridimensional (ux , uy , uz ) visto desde el sistema de referencia O. Supongamos que
el sistema Ō se mueve respecto a O con velocidad v en la dirección x.
50 CAPÍTULO 1. RELATIVIDAD ESPECIAL

a) Muestre que la velocidad del objeto (ux̄ , uȳ , uz̄ ) visto desde Ō está dada por:
ux v uy uz
ux̄ = , uȳ = , uz̄ =
1 ux v (1 ux v) (1 ux v)

b) Muestre, además, que si la magnitud de la velocidad tridimensional se define


de la manera usual u2 = (ux )2 + (uy )2 + (uz )2 , entonces se cumple que

(1 v 2 )(u2 1)
ū2 1=
(1 ux v)

c) Finalmente, muestre que si |v| < 1 y |u| < 1 entonces |ū| < 1, y que si |u| = 1
entonces |ū| = 1 para toda v.

1.14. Efectos de proyección. En ocasiones, efectos de proyección pueden ocasionar que


ciertos objetos astrofı́sicos se muevan aparentemente más rápido que la luz. Para
ver esto, considere un cuásar que emite hacia nosotros una nube de gas que viaja
con velocidad v < 1, a un ángulo ✓ con respecto a la lı́nea que une al cuásar con el
observador. Muestre que si el cuásar es muy lejano a nosotros, la velocidad tangencial
aparente de la nube resulta estar dada por Vt = v sin ✓/(1 v cos ✓). Si el ángulo ✓ es
muy pequeño (✓ ⌧ 1), muestre que la velocidad tangencial aparente Vt será mayor
que la de la luz siempre que v > 1/(1 + ✓). (Hint: considere dos mediciones de la
posición de la nube a tiempos t1 y t2 y recuerde que, como la nube se mueve hacia
nosotros, la distancia que la luz debe viajar para llegar al observador es cada vez
menor.)

También podría gustarte